You are on page 1of 62

lOMoARcPSD|5612816

Taxation Quizzer - QUIZZES FOR TAX

Bachelor of Science and Accoutancy (University of San Carlos)

StuDocu is not sponsored or endorsed by any college or university


Downloaded by jaene gallora (nichjaene05@gmail.com)
lOMoARcPSD|5612816

TAXATION QUIZZER PART 1

BASIC PRINCIPLES OF TAXATION

1. Which theory in taxation states that without taxes, a government


would be paralyzed for lack of power to activate and operate it,
resulting in its destruction?
a. Power to destroy theory
b. Lifeblood theory
c. Sumptuary theory
d. Symbiotic doctrine

2. The actual effort exerted by the government to effect the exaction


of what is due from the taxpayer is known as
a. Assessment.
b. Levy.
c. Payment.
d. Collection.

3. Although the power of taxation is basically legislative in


character, it is NOT the function of Congress to
a. Fix with certainty the amount of taxes.
b. Collect the tax levied under the law.
c. Identify who should collect the tax.
d. Determine who should be subject to the tax.

4. An example of a tax where the concept of progressivity finds


application is the
a. Income tax on individuals.
b. Excise tax on petroleum products.
c. Value-added tax on certain articles.
d. Amusement tax on boxing exhibitions.

5. Ligaya Educational Foundation, Inc., a stock educational


institution organized for profit, decided to lease for commercial
use a 1,500 sq. m. portion of its school. The school actually,
directly, and exclusively use the rents for the maintenance of its
school buildings, including payment of janitorial services. Is the
leased portion subject to real property tax?
a. Yes, since Lualhati is a stock and for profit educational
institution.
b. No, since the school actually, directly, and exclusively
used the rents for educational purposes.
c. No, but it may be subject to income taxation on the rents it
receives.
d. Yes, since the leased portion is not actually, directly, and
exclusively used for educational purposes.

6. Which among the following concepts of taxation is the basis for


the situs of income taxation?
a. Lifeblood doctrine of taxation
b. Symbiotic relation in taxation

Downloaded by jaene gallora (nichjaene05@gmail.com)


lOMoARcPSD|5612816

c. Compensatory purpose of taxation


d. Sumptuary purpose of taxation

DONOR’S TAX
7. Which of the following transactions is deemed a taxable gift?
a. Condonation or remission of a debt
b. Sale of residential house and lot for less than adequate and
full consideration in money or money’s worth
c. Both (a) and (b)
d. Neither (a) nor (b)

8. Which of the following statements relative to donor’s tax is


false?
a. The spouses shall file separate donor’s tax returns where
the thing donated is common property.
b. Each parent shall be entitled to the P10,000 exemption on
account of marriage of a child.
c. Exemptions and deductions cannot be claimed where the 30%
tax rate on stranger is applicable.
d. None of the foregoing.

9. The spouses Esme and Carlisle wanted to donate a parcel of land to


their son Edward who is getting married in December, 2016. The
parcel of land has a zonal valuation of P420,000.00. What is the
most efficient mode of donating the property?
a. The spouses should first donate in 2016 a portion of the
property valued at P20,000, then spread the P400,000 equally
for 2017, 2018, 2019 and 2020.
b. Spread the donation over a period of 5 years by the spouses
donating P100,000 each year from 2016 to 2020.
c. The spouses should each donate a P110,000 portion of the
value of the property in 2016 then each should donate
P100,000 in 2017.
d. The spouses should each donate a P100,000 portion of the
value of the property in 2016, and another P100,000 each in
2017. Then, in 2018, Esme should donate the remaining
P20,000.

10. Exempted from donor’s taxation are gifts made


a. For the use of the barangay.
b. In consideration of marriage.
c. To a school which is a stock corporation.
d. To a for-profit government corporation.

11. Caroline donated P110,000.00 to her friend Vicky who was getting
married. Caroline gave no other gift during the calendar year.
What is the donor's tax implication on Caroline’s donation?
a. The P100,000 portion of the donation is exempt since given
in consideration of marriage.
b. A P10,000 portion of the donation is exempt being a donation
in consideration of marriage.

Downloaded by jaene gallora (nichjaene05@gmail.com)


lOMoARcPSD|5612816

c. Caroline shall pay a 30% donor's tax on the P110,000


donation.
d. The P100,000.00 portion of the donation is exempt under the
rate schedule for donor's tax.

12. A non-stock, non-profit school always had cash flow problems,


resulting in failure to recruit well-trained administrative
personnel to effectively manage the school. In 2017, Don Leon
donated P100 million pesos to the school, provided the money shall
be used solely for paying the salaries, wages, and benefits of
administrative personnel. The donation represents less than 10% of
Don Leon's taxable income for the year. Is he subject to donor's
taxes?
a. No, since the donation is actually, directly, and
exclusively used for educational purposes.
b. Yes, because the donation is to be wholly used for
administration purposes.
c. Yes, since he did not obtain the requisite NGO certification
before he made the donation.
d. No, because the donation does not exceed 10% of his taxable
income for 2017.

13. What law shall govern the imposition of donor’s tax?


a. The law in force at the time of perfection of the donation
b. The law in force at the time of completion of the donation
c. The law in force at the time of perfection or completion
depending upon the agreement of the parties
d. None of the choices

14. Andy, married, donated a land commonly owned by him and her spouse
worth P500,000 to her friend Joan. Only Andy signed the deed of
donation. Joan assumed P200,000 unpaid mortgage on the property.
How much is the donor’s tax due?
a. P6,000 c. P1,000
b. P90,000 d. P45,000

500k - 200k = 300k x 30% = 90,000

15. Dondie, resident citizen, made the following donations on April 28


of the current year:
 To his sister, Donna,P175,000 worth of property situated in
Paris, France. The donor’s tax paid is P40,000.
 To Dara, his girlfriend in the Philippines, jewelry valued at
P225,000.
 To International Rice Research Institute, cash amounting to
P50,000.
The donor’s tax due after tax credit is –
a. P69,000 c. P29,000
b. P38,813 d. P5,625

Downloaded by jaene gallora (nichjaene05@gmail.com)


lOMoARcPSD|5612816

225,000 x 30% = 67,500 + (175,000 - 100,000 x 2% Tabular) = 69,000 -


(69,000 x 175k/400k) = 38,813

ESTATE TAX
16. Which of the following is not a part of the gross estate?
a. Conjugal property
b. Community property
c. Exclusive property of the decedent
d. Exclusive property of the surviving spouse

17. Who among the following transferors is not liable for estate tax
on the property transferred during his lifetime?
a. The testator who bequeaths property to his heirs in a last
will and testament executed and probated during his lifetime
b. The donor who reserves his right to amend or revoke the
donation of property in favor of the donee
c. The donee of an appointed property who is required under a
power of appointment to transfer such property upon death to
his eldest child
d. The transferor of personal property who sold it for
insufficient consideration

18. Which of the following properties of the spouses will be part of


common properties under a regime of Conjugal Partnership of Gains?
a. Land inherited during the marriage
b. Fruits of land inherited
c. Jewelry inherited during the marriage
d. Building donated before marriage

19. Which of the following items is not considered as a “special


deduction” in computing the taxable net estate of the decedent?
a. Vanishing deduction
b. Medical expenses
c. Standard deduction
d. Family home allowance

20. When the payment of estate tax will cause undue hardship upon the
heirs or the estate which is undergoing judicial settlement before
the court, the BIR Commissioner may grant an extension for a
period not exceeding:
a. 5 years
b. 3 years
c. 2 years
d. 1 year

Next three (3) questions are based on the following:


Orland, married resident citizen, died on August 20, 2016. The estate
reported the following assets and deductions:
Conjugal Properties:
Fishpond, Bulacan P1,500,000
Family Home, Makati 1,500,000
Cash in bank 900,000

Downloaded by jaene gallora (nichjaene05@gmail.com)


lOMoARcPSD|5612816

Exclusive Properties of Orland:


Land, inherited from his father who P400,000
died on July 20, 2012. The value of
land at the time of inheritance was
P210,000. The land was mortgaged for
P30,000 which was unpaid at the time of
death of his father,P10,000 of which
was paid by Orland before he died.

Land, donated on February 14, 2013 by 600,000


his mother who died on November 2,
2014. The value of the land when
donated was P500,000 while upon death
of his mother was P400,000.

Exclusive Properties of Wife:


Farm in Laguna, acquired before 2,000,000
marriage

Deductions claimed:
Funeral expenses 250,000
Fire loss of apartment (occurred 4 80,000
months after death)
Bad debts (represents unpaid receivable 100,000
from Bert, an insolvent)
Mortgage on inherited land 30,000
Vanishing deduction on inherited 40,000
land
Vanishing deduction on donated land 20,000
Standard deduction 2,000,000

21. The gross estate is:


a. P4,000,000 c. P7,000,000
b. P4,710,000 d. P5,000,000

22. The vanishing deduction is:


a. P184,000 c. P255,760
b. P220,800 d. P292,560

23. The net taxable estate is:


a. P819,200 c. P829,200
b. P804,200 d. P579,200

Downloaded by jaene gallora (nichjaene05@gmail.com)


lOMoARcPSD|5612816

SOLUTION:
Properties Exclusive Conjugal Total
Fishpond, Bulacan P1,500,000 1,500,000
Family Home, Makati 1,500,000 1,500,000
Cash in bank 900,000 900,000
Land, inherited from Father
died on July 20, 2012. 400,000 400,000
Land, Donated from Mother
on Feb 14, 2013. 600,000 600,000
Claims to Insolvent 100,000 100,000
GROSS ESTATE 1,000,000 4,000,000 5,000,000
Allowable Deductions:
Funeral Expenses (200,000) (200,000)
Fire Loss (80,000) (80,000)
Bad Debts (100,000) (100,000)
Mortgage on Land (20,000) (20,000)
Vanishing Deductions* (220,800) (220,800)
Net Estate before Special
Deductions 759,200 3,620,000 4,379,200
Special Deductions:
Family Home (1,500,000 x 1/2) (750,000)
Special Deduction (1,000,000)
Share of Surviving Spouse (1,810,000)
NET TAXABLE ESTATE 819,200

*VANISHING DEDUCTIONS:
Land Inherited by Father:
Value at the time of Death 210,000
Less: Mortgage Paid (10,000)
Initial Basis 200,000
Pro rata: 200/5000 x 400,000 (16,000)
Final Basis 184,000
Vanishing Rate (4 years but not more 5 yrs) 20%
Vanishing Deduction 36,800

Land Donated by Mother:


Value at the time of Donation 500,000
Initial Basis 500,000
Pro rata: 500/5000 x 400,000 (40,000)
Final Basis 460,000
Vanishing Rate (3 years but not more 4 yrs) x 40%
Vanishing Deduction 184,000

TOTAL VANISHING DEDUCTIONS 220,800

**If TRAIN Law is applied the Estate tax is P 49,152 (P819,200 x 6%)

Downloaded by jaene gallora (nichjaene05@gmail.com)


lOMoARcPSD|5612816

24. Abe, married resident alien, died on January 15, 2017. She left
the following properties, expenses and obligations:

Community properties, Philippines (including P5,000,000


family home valued at P1,800,000)
Community properties, Abroad 2,000,000
Exclusive properties, Philippines 3,000,000
Actual funeral expenses 300,000
Judicial expenses 200,000
Medical expenses (incurred w/in 1yr. before death) 600,000
Devise to National Gov’t 50,000
Legacy to Local Gov’t 70,000

The net taxable estate is:


a. P3,780,000 c. P3,580,000
b. P3,680,000 d. P3,530,000

RESIDENT ALIEN
Particulars Exclusive Community Total
All Properties w/i & w/o 3,000,000 7,000,000 10,000,000
Funeral Expense (200,000) (200,000)
Judicial Expense (200,000) (200,000)
Transfers (50,000 + 70,000) (120,000) (120,000)
Gross Estate 2,880,000 6,600,000 9,480,000
Share of Surviving Spouse (3,300,000)
Medical Expenses (500,000)
Family Home (1/2 of 1,800,000) (900,000)
Standard Deductions (1,000,000)
Net Estate 3,780,000

25. Based on the above problem, if the decedent is a non-resident


alien, how much is the net taxable estate?
a. P2,755,000 c. P5,220,000
b. P2,880,000 d. P5,380,000

NON - RESIDENT ALIEN


Particulars Exclusive Community Total
All Properties w/i Only 3,000,000 5,000,000 8,000,000
Funeral Expense 200,000
Judicial Expense 200,000
Allowable Deduction 400,000 x 8M/10M (320,000) (320,000)
Transfers(50,000 + 70,000) (120,000) (120,000)
Gross Estate 2,880,000 4,680,000 7,560,000
Share of Surviving Spouse (2,340,000)
Net Estate 5,220,000

26. Arthur, Filipino, married died leaving the following estate:

Car acquired before marriage by Arthur P 300,000

Downloaded by jaene gallora (nichjaene05@gmail.com)


lOMoARcPSD|5612816

Car acquired before marriage by wife 450,000


House and lot acquired during marriage 1,500,000
Jewelries of wife 100,000
Personal properties inherited by Arthur during 250,000
marriage
Benefits from SSS 50,000
Retirement benefits 150,000
Proceeds of group insurance taken by his employer 75,000
Land inherited by the wife during marriage 1,000,000
Income earned from the land inherited by wife (25% 200,000
of which was earned after death)

16. How much is the gross estate if the property relationship is


conjugal partnership of gains is:
a. P2,600,000 c. P1,950,000
b. P3,600,000 d. P2,200,000

Car acquired before marriage by Arthur P 300,000


House and lot acquired during marriage 1,500,000
Personal properties inherited by Arthur during marriage 250,000
Income earned from the land inherited by wife
(25% of which was earned after death) 150,000
Gross Estate 2,200,000

27. Based on the preceding number, the gross estate if the property
relationship is absolute community of property is:
a. P2,600,000 c. P1,950,000
b. P3,600,000 d. P2,500,000

Car acquired before marriage by Arthur P 300,000


Car acquired before marriage by wife 450,000
House and lot acquired during marriage 1,500,000
Jewelries of wife 100,000
Personal properties inherited by Arthur during marriage 250,000
Gross Estate 2,600,000

VAT

28. LBJ made the following sales during the 12-month period:

Sales, VAT taxable transactions P1,500,000


Sales, VAT zero-rated transactions 400,000
Sales, VAT exempt transactions 100,000
Total P2,000,000

Which of the following statements is correct?


a. LBJ may not register under the VAT system because his sales from
VAT taxable transactions did not exceed P1,919,500.
b. LBJ may not register under the VAT system because his sales from
VAT taxable and zero-rated transactions did not exceed
P1,919,500.

Downloaded by jaene gallora (nichjaene05@gmail.com)


lOMoARcPSD|5612816

c. LBJ is required to register because his total 12-month sales


exceeded P1,919,500.
d. None of the foregoing.

29. Which of the following is exempt from VAT?


a. Common carriers transporting passengers by air within the
Philippines
b. Common carriers transporting passengers by sea within the
Philippines
c. Common carriers transporting passengers by land within the
Philippines
d. Common carriers transporting cargoes by air within the
Philippines

30. Which statement is correct about value-added tax on goods or


properties sold?
a. It is based on gross sales and not on net sales;
b. May be due even if the goods or properties were not actually
sold;
c. Does not cover goods exported;
d. It forms part of the selling expense of the trader.

31. For value-added tax purposes, which of the following transactions


of a VAT-registered taxpayer may not be zero-rated?
a. Export sales
b. Foreign currency denominated sales
c. Sale of goods to the Asian Development Bank
d. Sale of goods to an export oriented enterprise

32. A subdivision developer sold five (5) residential house and lots,
each to different vendees, for P3,000,000 per lot, or a total
sales of P15,000,000 for the taxable period.
These sales shall be classified as:
a. 12% VAT transactions
b. 0% VAT transactions
c. VAT exempt transactions
d. None of the foregoing

**3,199,000 each is the threshold

33. CP operated a retail business that had been generating sales not
exceeding the threshold for VAT exempt persons. However, he
desires to be registered under the VAT system for the first time
in order to benefit from input tax credits.
What benefit may CP be entitled to once he registers under the VAT
system?
a. Tax refund
b. Presumptive input tax credit
c. Transitional input tax credit
d. None of the foregoing

Downloaded by jaene gallora (nichjaene05@gmail.com)


lOMoARcPSD|5612816

34. What institution is required to deduct and withhold a final VAT of


5% on the purchase of goods or services subject to VAT?
a. National government or any political subdivision thereof
b. Government-owned or controlled corporations
c. Both (a) and (b)
d. Neither (a) nor (b)

35. In the value-added tax on sale of services, the output tax is


computed:
a. On the billings of the month
b. On collections of the month on all billings made
c. On the contract price of contracts completed during the
taxable period
d. Only and strictly on labor performed under the contract for
services

36. Which statement is wrong?


a. There is a transitional input tax from purchases of goods or
properties;
b. There is a transitional input tax from purchases of
services;
c. There is a transitional input tax from purchases of
materials;
d. There is a transitional input tax from purchases of
supplies.

37. Which of the following statements is correct on the inventory


balance in the financial statements at any given date of a VAT-
registered person?
a. Balance, net of input taxes
b. Balance, inclusive of input taxes
c. Balance on which the transitional input tax is computed
annually
d. Balance where the VAT thereon may be calculated by
multiplying it by 12%

38. Genson Distribution Inc., a VAT taxpayer, had the following data
in a month:

Cash sales P200,000


Open account sales 500,000
Consignment:
0 to 30 days old (on which there
were remittances from consignees of
P200,000) 600,000
31 to 60 days old 700,000
61 days old and above 900,000
How much is the output tax?
a. P348,000 c. P264,000
b. P216,000 d. P108,000

Downloaded by jaene gallora (nichjaene05@gmail.com)


lOMoARcPSD|5612816

Cash sales P200,000


Open account sales 500,000
Consignment:
0 to 30 days old (on which there were
remittances from consignees of P200,000) 200,000
61 days old and above 900,000
Total VATABLE SALES 1,800,000
VAT RATE 12%
OUTPUT VAT 216,000

42. The financial records of Benz Corp., a VAT-registered taxpayer,


for the taxable year 2016 disclosed the following:
Local sales to private entities 1,500,000
Export Sales 500,000
Local sales to government 800,000

How much is the total sales subject to value-added tax?


a. P2,800,000 c. P2,000,000
b. P2,300,000 d. P1,500,000

Local sales to private entities 1,500,000


Export Sales 500,000
Local sales to government 800,000
Total VATABLE SALES 2,800,000

43. Mantika Corp., a VAT-registered Corp., is a producer of cooking


oil from coconut and corn. It had the following data for the month
of January 2017:
Sales, gross of VAT P 784,000
Corn & Coconut, 12-31-16 50,000
Purchases of Corn & Coconut 330,000
Corn & Coconut, 1-31-17 20,000
Purchases from VAT suppliers, VAT included:
Packaging Materials 56,000
Supplies 16,800
The value-added tax payable for the month:
a. P56,060 c. P60,650
b. P54,900 d. P63,000
Sales, gross of VAT P 784,000
Output TAX 84,000

Purchases of Corn & Coconut (330,000 x4%) (13,200)


Purchases from VAT suppliers, VAT included:
Packaging Materials 56,000
Supplies 16,800 72,800 x3/28 (7,800)
VAT PAYABLE 63,000

Downloaded by jaene gallora (nichjaene05@gmail.com)


lOMoARcPSD|5612816

44. Bunga Inc., a VAT taxpayer, is engage in the business of


processing of fruits. Its data on sales and purchases for the
month of August are provided below:
Sales P200,000
Purchases:
Fresh Fruits 30,000
Raw sugarcane 12,000
Tin Can, gross of VAT 12,320
Paper Labels, net of VAT 5,000
Cardboard for boxes, net of VAT 8,000
Freight, gross of VAT (50% still unpaid) 10,080

How much is the value-added tax payable?


a. P20,580 c. P19,380
b. P18,900 d. P20,100

Sales P200,000 Output Tax 24,000


Purchases:
Fresh Fruits 30,000
Raw sugarcane 12,000
Tin Can, gross of VAT 12,320 Input Tax (1,320)
Paper Labels, net of VAT 5,000 Input Tax ( 600)
Cardboard for boxes, net of VAT 8,000 Input Tax ( 960)
Freight, gross of VAT
(50% still unpaid) 10,080 Input Tax ( 540)
VAT PAYABLE 20,580

45. Bahay Kubo Inc. is a real estate dealer. Details of its sales
during the year showed the following:
Date of sale June 2, 2017
Consideration in the deed of sale P 5,000,000
Fair market value in the assessment rolls 4,800,000
Zonal Value 5,200,000

Schedule of payments:
June 2, 2017 1,000,000
June 2, 2018 2,000,000
June 2, 2019 2,000,000

How much is the output tax to be recognized for the June 2, 2018
payment?
a. P0 c. P249,600
b. P124,800 d. P624,000

**Zonal Value 5,200,000 x 12% = 624,000 x 2M/5M =


249,600 Output Tax for 2018

46. Assuming that the scheduled payment on June 2, 2017 is P2,000,000,


how much is the output tax to be recognized for the June 2, 2019
payment?
a. P0 c. P249,600
b. P124,800 d. P624,000

Downloaded by jaene gallora (nichjaene05@gmail.com)


lOMoARcPSD|5612816

Zero as in 0 for the sale will no longer qualify as Installment


Sales.

47. Mr. Karpentero, a vat-registered building contractor, has the


following data on gross receipts in a month, any tax not included:

 From Mr. A, a private property owner, final payment on the


contract price, net of 5% agreed retention fee of P2,850,000

 From Mr. B, a payment of 5% retention on the contract price


previously made by him P100,000

 From Mr. C, for materials used in the construction 500,000

How much is the output tax?


a. P414,000 c. P72,000
b. P342,000 d. P62,000

Final Payment on Contracts 2,850,000


Retention 100,000
Materials 500,000
TOTAL Receipts 3,450,000
VAT Rate 12%
Output VAT 414,000

48. COC Inc., in its first month of operation, and as a VAT taxpayer,
purchased various fixed assets. Purchases of fixed assets in the
first month were as follows:

Light equipment, with a useful life of 3 years P 300,000


Heavy equipment, with a useful life of 10 years 4,000,000

How much is the input tax available for the month?


a. P516,000 c. P480,000
b. P9,000 d. P8,600

Light Equipment 300,000/36 x 12% 1,000


Heavy Equipment 4,000,000/60 x 12% 8,000
Total Input VAT 9,000

49. Kusina Co., had its kitchen assembled by a VAT taxpayer. It took
six months for the contractor to finish the work. Kusina Co.
purchased materials in July from VAT suppliers at a cost of
P500,000, VAT not included. Payment to the contractor in July 2017
on the Construction in Progress, VAT not included was:

On contractor’s billing in June P100,000


On contractor’s billing in July 70,000

Downloaded by jaene gallora (nichjaene05@gmail.com)


lOMoARcPSD|5612816

The input tax available in July is:


a. P0 c. P60,000
b. P80,400 d. P20,400

Materials from VAT Supplier 500,000


On contractor’s billing in June 100,000
On contractor’s billing in July 70,000
Total 670,000
VAT Rate 12%
Output Tax 80,400

50. Data from the books of accounts of a VAT taxpayer for February:
Domestic Exports
Sales P 2,000,000 8,000,000
Purchases:
From VAT Suppliers:
Goods for sale 600,000 2,400,000
Supplies & services 90,000 360,000
From Suppliers Paying percentage tax:
Goods for sale 100,000 1,500,000
Supplies & services 20,000 80,000
If the input taxes attributable to zero-rated sales are claimed as
tax credit, the net value-added tax refundable is:
a. P136,000 c. P145,000
b. P203,924.70 d. P174,000

Output Tax (2,000,000 x 12%) 240,000


Input Tax on Domestic Sales (690,000 x 12%) (82,800)
Input Tax on Zero Rated (2,760,000 x 12%) (331,200)
Refundable VAT (174,000)

INCOME TAXATION
51. C. Lee, Chinese national, arrived in the Philippines on January 1,
2012 to visit his Filipina paramour. He planned to stay in the
country until December 31, 2016, by which time he would go back to
his legal wife and family in China. C. Lee derived income during
his stay here in the Philippines.
For the taxable year 2012, C. Lee shall be classified as a:
a. Resident alien
b. Non-resident alien engaged in trade or business in the
Philippines
c. Non-resident alien not engaged in trade or business in the
Philippines
d. Special alien employee

52. The following individual taxpayers are subject to the graduated


income tax rates of 5%-32%, except
a. Filipino citizens
b. Resident aliens
c. Non-resident alien engaged in trade or business in the
Philippines

Downloaded by jaene gallora (nichjaene05@gmail.com)


lOMoARcPSD|5612816

d. Non-resident alien not engaged in trade or business in the


Philippines

53. In which of the following cases will the dividend income from a
foreign corporation be classified as “income without”
a. Less than 50% of the foreign company’s gross income for the
preceding three (3) years prior to the dividend declaration
was derived from sources within the Philippines.
b. 50% of the foreign company’s gross income for the preceding
three (3) years prior to the dividend declaration was
derived from sources within the Philippines.
c. More than 50% of the foreign company’s gross income for the
preceding three (3) years prior to the dividend declaration
was derived from sources within the Philippines.
d. Always classified as income without”.

54. D’ Lion, Inc., a Philippine corporation, sold through the local


stock exchange 10,000 PLDT shares that it bought 2 years ago. D’
Lion sold the shares for P2 million and realized a net gain of
P200,000. How shall it pay tax on the transaction?
a. It shall declare a P2 million gross income in its income tax
return, deducting its cost of acquisition as an expense.
b. It shall report the P200,000 in its corporate income tax
return adjusted by the holding period.
c. It shall pay 5% tax on the first P100,000 of the P200,000
and 10% tax on the remaining P100,000.
d. It shall pay a tax of one-half of 1% of the P2 million gross
sales.

55. The payor of passive income subject to final tax is required to


withhold the tax from the payment due the recipient. The
withholding of the tax has the effect of
a. A final settlement of the tax liability on the income.
b. A credit from the recipient's income tax liability.
c. Consummating the transaction resulting in an income.
d. A deduction in the recipient's income tax return.

56. Winterfell, Inc., bought a parcel of land in 2015 for P7 million


as part of its inventory of real properties. In 2017, it sold the
land for P12 million which was its zonal valuation. In the same
year, it incurred a loss of P6 million for selling another parcel
of land in its inventory. These were the only transactions it had
in its real estate business. Which of the following is the
applicable tax treatment?
a. Winterfell shall be subject to a tax of 6% of P12 million.
b. Winterfell could deduct its P6 million loss from its P5
million gain.
c. Winterfell’s gain of P5 million shall be subject to the
holding period.
d. Winterfell's P6 million loss could not be deducted from its
P5 million gain.

Downloaded by jaene gallora (nichjaene05@gmail.com)


lOMoARcPSD|5612816

57. Passive income includes income derived from an activity in which


the earner does not have any substantial participation. This type
of income is
a. Usually subject to a final tax.
b. Exempt from income taxation.
c. Taxable only if earned by a citizen.
d. Included in the income tax return.

58. In 2017, Alice earned P500,000 as income from her beauty parlor
and received P250,000 as Christmas gift from her aunt. She had no
other receipts for the year. She spent P150,000 for the operation
of her beauty parlor. For tax purposes, her gross income for 2017
is
a. P750,000 c. P350,000
b. P500,000 d. P600,000

59. Which of the following items is not part of gross income to be


reported in the income tax return?
a. Increase in value of land
b. Gambling winnings
c. Prize of P10,000
d. Gain from sale of store’s air conditioner

60. Mr. Yu leased his lot to Mr. Uy. The contract calls for Mr. Uy to
construct a house which would serve as the residence of the
latter, the ownership thereof to be vested in Mr. Yu after the
expiration of the lease. When the house was completely
constructed, the remaining term of the lease was 10 years. The
residential house had an estimated useful life of 15 years.
What is the tax implication of the leasehold improvement?
a. Mr. Yu derives taxable income on the improvement; Mr. Uy can
claim depreciation expense as a deduction from gross income.
b. Mr. Yu derives taxable income on the improvement; Mr. Uy
cannot claim depreciation expense as a deduction from gross
income.
c. Mr. Yu does not derive taxable income on the improvement;
Mr. Uy cannot claim depreciation expense as a deduction from
gross income.
d. Mr. Yu does not derive taxable income on the improvement;
Mr. Uy can claim depreciation expense as a deduction from
gross income.

61. Assume the same facts in the immediately preceding number, except
that at the time of the completion of the residential house, the
remaining term of the lease was 15 years while the useful life of
the house was 10 years.
What is the tax implication of the leasehold improvement?
a. Mr. Yu derives taxable income on the improvement; Mr. Uy can
claim depreciation expense as a deduction from gross income.
b. Mr. Yu derives taxable income on the improvement; Mr. Uy
cannot claim depreciation expense as a deduction from gross
income.

Downloaded by jaene gallora (nichjaene05@gmail.com)


lOMoARcPSD|5612816

c. Mr. Yu does not derive taxable income on the improvement;


Mr. Uy cannot claim depreciation expense as a deduction from
gross income.
d. Mr. Yu does not derive taxable income on the improvement;
Mr. Uy can claim depreciation expense as a deduction from
gross income.

62. Which of the following expenses may be deducted from gross


compensation income?
a. Depreciation of permanent assets
b. Premium payments on health and/or hospitalization insurance
c. Bad debts written off
d. Optional standard deduction

63. Which of the following items of interest expense may be deducted


from gross income?
a. Interest on corporation’s preferred stock
b. Interest on loan for construction of a rest house
c. Interest for delinquency in the payment of percentage tax
d. Interest on bank loan to finance petroleum exploration

64. Which of the following taxes may be deducted from gross income?
a. Percentage tax on sale of listed stock
b. Business permit fee paid to the city government
c. Income tax
d. Tax on interest on bank deposit

65. Who among the following taxpayers may not claim a tax credit or
deduction on income tax paid to foreign countries?
a. Resident citizens
b. Resident aliens
c. Domestic corporations
d. General Co-Partnerships

66. The loss from sale or exchange of property is deductible from


gross income where the sale or exchange is:
a. Between fiduciary of a trust and the fiduciary of another
trust if they have the same grantor
b. Between fiduciary of a trust and the beneficiary of such
trust
c. Between an individual and his first cousin
d. Between an individual and a corporation if the former owns
more than 50% in value of the outstanding capital stock of
the latter

67. Anne, claimed a bad debt of P50,000 as a deductible expense in the


taxable year 2016. In 2017, Anne was able to recover the P50,000
debt already written off in the preceding year.
What is the treatment for tax purposes of the recovery of the bad
debt?
a. Report the recovery of the bad debt as gross income in 2016.
b. Report the recovery of the bad debt as gross income in 2017.

Downloaded by jaene gallora (nichjaene05@gmail.com)


lOMoARcPSD|5612816

c. Disregard the recovery of the bad debt.


d. Amend the 2016 income tax return to rectify the deduction
for bad debt claimed.

68. Which of the following assets shall be subject to depletion?


a. Machinery
b. Land containing ore deposit
c. Commercial
d. Goodwill

69. Which of the following individual taxpayers may claim basic and
additional personal exemptions for income tax purposes?
a. Non-resident aliens engaged in trade or business in the
Philippines, in the absence of reciprocity
b. Non-resident aliens not engaged in trade or business in the
Philippines
c. Both (a) and (b)
d. Neither (a) nor (b)

70. Harry works as financial consultant in an oil firm in Dubai. Aside


from his salary thereat, he also maintains a 10-door apartment in
Manila which he inherited from his parents when he was already
married. On the other hand, Wilma, his wife, is employed as a loan
officer at a local bank. Data pertaining to their dependents
appear below for the taxable year 2017:

 Anton - Son who turned 23 on April 1, 2017; incapable of


self-support due to loss of both legs in an accident;
 Bunny - 21 year old daughter who is taking up culinary arts
in Paris, France;
 Charlie - 15-year old adulterous son of Harry living with the
couple;
 Dina - 12-year old child who died from a vehicular accident
on January 1, 2017; and
 Evan - 80-year old father of Wilma, supported by her and
living with the couple.

The basic and additional personal exemptions of Harry for the


taxable year 2017 amounts to:
a. P50,000 and P100,000, respectively
b. P50,000 and P75,000, respectively
c. P50,000 and P0, respectively
d. P0 and P0, respectively

71. Assume the same facts above, the basic and additional personal
exemptions of Wilma for the taxable year 2016 amount to:
a. P50,000 and P100,000, respectively
b. P50,000 and P75,000, respectively
c. P50,000 and P0, respectively
d. P0 and P0, respectively

Downloaded by jaene gallora (nichjaene05@gmail.com)


lOMoARcPSD|5612816

72. Which of the following statements does not characterize a capital


asset?
a. It may be real or personal property.
b. It is not always subject to a holding period.
c. It is normally subject to value-added tax when it is sold.
d. It is not always subject to a final tax.

73. Which of the following transactions is exempt from capital gains


tax?
a. The sale of the principal residence of the taxpayer where
the entire proceeds is used to purchase a vacation lot at
Tagaytay
b. The sale of a beach lot of the taxpayer where the entire
proceeds is used to construct his principal residence
c. The sale of the principal residence of the taxpayer for the
second time in ten (10) years to purchase another principal
residence
d. None of the choices

74. Which of the following transactions is treated as a capital asset


transaction for income tax purposes?
a. Sale of a residential lot by a subdivision developer
b. Sale of a used delivery truck by a retailing company
c. Liquidation of partnership business
d. Sale of shares of stock by a dealer in securities

75. Which of the following is not an attribute of a deferred-payment


sale?
a. The initial payments exceed 25% of the selling price in the
year of sale.
b. The obligations or promissory notes received by the vendor
from the vendee are considered as equivalent to cash.
c. The tax may be paid in installments.
d. The sale involves both real and personal property.

76. The deductible expenses of an estate may consist of:


a. Deductible expenses allowed to an individual taxpayer
b. Income distributed to beneficiaries
c. Both (a) and (b)
d. Neither (a) nor (b)

77. Determine which of the following trusts shall the taxable income
be consolidated and the income tax thereon computed on the basis
of such consolidated income?
a. Trust No. 1 and Trust No. 2 have the same grantor and with
different beneficiaries.
b. Trust No. 1 and Trust No. 2 have the same grantor and the
same beneficiary.
c. Trust No. 1 and Trust No. 2 have different grantors and the
same beneficiary.
d. Trust No. 1 and Trust No. 2 have different grantors and the
same fiduciary and beneficiary.

Downloaded by jaene gallora (nichjaene05@gmail.com)


lOMoARcPSD|5612816

78. Inday is a resident citizen of the Philippines. Data for a year:


Gross income from business P 700,000
Royalty from books 40,000
Gain on direct sale to buyer of
shares of stock of a domestic corporation
held as capital asset 70,000
Loss on sale of land in the Philippines held
as capital asset with cost of P1,500,000 when the
zonal value is P1,200,000 500,000
Business Expenses 300,000
How much is the total income tax expense for the year?
a. P177,500 c. P159,500
b. P80,000 d. P156,000

Gross Income from Business 700,000


Less: Business Expense 300,000
Personal Exemptions 50,000 350,000
Net Taxable income 350,000

Tabular Schedule:
1st 250,000 Tax is 50,000
Excess of 250,000 - 350,000 = 100,000 x 30% 30,000
Capital Gains Tax on Shares (70,000 x 5%) 3,500
Royalty Income (40,000 x 10%) 4,000
Loss on Sale (Zonal Value 1.2 x 6%) 72,000
Total Income Tax Expense 159,500

79. Mercy is a citizen and resident of the Philippines. She had a


compensation income (net of exclusions) of P200,000 and a net
income from business of P700,000 for a year. She made quarterly
income tax payments amounting to P237,000 and her employer
withheld P25,000 on her compensation income.
The income tax payable (refundable) for the year is:
a. (P25,000) c. (P42,000)
b. P237,000 d. (P37,500)

Compensation Income 200,000


Net Income from Business 700,000
Total Income 900,000
Basic Exemptions (50,000)
Taxable Income 850,000
1st 500,000 Tax 125,000
Excess of 500,000- 850,000 x 32% 112,000
Total Tax Due 237,000
Less: Taxes Payments
Quarterly Payments 237,000
Compensation CWT 25,000 262,000
Refundable /Creditable Tax (25,000)

Downloaded by jaene gallora (nichjaene05@gmail.com)


lOMoARcPSD|5612816

80. EMT has the following data on his passive income earned during the
year 2016:
Philippines Abroad
Interest income from bank deposits 45,000 25,000
Interest income from FCDU 50,000 -0-
Royalties from books 20,000 30,000
Royalties from computer programs 20,000 40,000
Dividend income from a domestic corporation 27,000 13,000
Dividend income from a foreign corporation 33,000 22,000

How much is the final withholding tax if the taxpayer is a


resident citizen?
a. P21,450 c. P17,700
b. P20,400 d. P36,250

Philippines
Interest income from bank deposits 45,000 x 20% = 9,000
Interest income from FCDU 50,000 x 7.5% = 3,750
Royalties from books 20,000 x 10% = 2,000
Royalties from computer programs 20,000 x 20% = 4,000
Dividend income from a
domestic corporation 27,000 x 10% = 2,700
Total Withholding Taxes 21,450

81. How much is the final withholding tax if the taxpayer is a


resident alien?
a. P21,450 c. P17,700
b. P20,400 d. P36,250
SAME COMPUTATION AS RESIDENT CITIZEN

82. How much is the final withholding tax if the taxpayer is a non-
resident citizen?
a. P21,450 c. P17,700
b. P20,400 d. P36,250
Philippines
Interest income from bank deposits 45,000 x 20% = 9,000
Royalties from books 20,000 x 10% = 2,000
Royalties from computer programs 20,000 x 20% = 4,000
Dividend income from a domestic
corporation 27,000 x 10% = 2,700
Total Withholding Taxes 17,700

83. How much is the final withholding tax if the taxpayer is a non-
resident alien engaged in trade or business?
a. P21,450 c. P17,700
b. P20,400 d. P36,250

Downloaded by jaene gallora (nichjaene05@gmail.com)


lOMoARcPSD|5612816

Philippines
Interest income from bank deposits 45,000 x 20% = 9,000
Royalties from books 20,000 x 10% = 2,000
Royalties from computer programs 20,000 x 20% = 4,000
Dividend income from a domestic
corporation 27,000 x 20% = 5,400
Total Withholding Taxes 20,400

84. How much is the final withholding tax if the taxpayer is a non-
resident alien not engaged in trade or business?
a. P21,450 c. P17,700
b. P20,400 d. P36,250

Interest income from bank deposits 45,000


Royalties from books 20,000
Royalties from computer programs 20,000
Dividend income from a domestic corporation 27,000
Dividend income from a foreign corporation 33,000
TOTAL PASSIVE INCOME 145,000
Final Withholding Tax Rate 25%
Total Withholding Taxes 36,250

85. Nonoy is an employee of a firm in Quezon City. He is supporting


his 4 year old brother who is living with him. Data on his
compensation income for the year shows:
Regular Salary P 240,000
Thirteenth month pay 20,000
Quarterly bonus 40,000
Payroll Deductions:
SSS Premiums 3,000
Philhealth contributions 1,200
Pagibig contributions 4,000
Labor union dues 1,000
Premium payments on hospitalization insurance 3,000
Payment of loan 5,000
86. How much is the taxable income?
a. P210,800 c. P208,400
b. P207,800 d. P260,800

Regular Salary P 240,000


Total Income 240,000
Less: Basic Exemption (50,000)
SSS Premiums (3,000)
Philhealth contributions (1,200)
Pagibig contributions (4,000)
Labor union dues (1,000)
Net Taxable Income 180,800 +30,000 if old laws = 218,000

Downloaded by jaene gallora (nichjaene05@gmail.com)


lOMoARcPSD|5612816

87. A domestic corporation, in its fifth (5th) year of operations, had


the following data for the year:
Net sales P 2,000,000

Capital gain on direct sale to a buyer


of shares of a domestic corporation for P500,000 200,000

Capital gain on sale thru a real


estate broker of land and building outside
the Philippines for P5,000,000 1,000,000

Dividend from a domestic corporation 50,000


Interest on bank deposit 40,000
Cost of sales 600,000
Quarterly corporate income tax paid 190,000
Operating expenses 500,000
The income tax payable upon filing of the annual income tax return
is:
a. P425,000 c. P570,800
b. P380,000 d. P520,800

Net Sales 2,000,000


Cost of Sales (600,000)
Operating Expenses (500,000)
Net Income from Ordinary Business 900,000
Add Other Income Broker Outside Phils 1,000,000
Total Taxable Income 1,900,000
Corporate Tax Rate 30%
Income Tax Due 570,000
Less Quarterly Income Tax Paid (190,000)
Income Tax Due and Payable 380,000

88. The Kultura Foundation of the Philippines, a non-stock, non-profit


corporation, organized and operated exclusively to preserve and
show-case Philippine cultural practices, music, dances, and folk
arts, deriving funding from mostly donations, had the following
data for the year:

Donations received P 20,000,000


Interest income from bank deposit 100,000
Rent income from properties received as donation
(net of 5% withholding tax) 475,000
Expenses related to its rent income 30,000

89. How much is the income tax expense of the Corporation for the
year?
a. P161,000 c. P172,000
b. P136,000 d. P125,600

Rent Income (Gross Up) 500,000


Less: Expenses (30,000)
Net Rent Income 470,000

Downloaded by jaene gallora (nichjaene05@gmail.com)


lOMoARcPSD|5612816

Tax Rate 30%


Tax Expense 141,000
Add Interest Income Tax 20,000
Total Income Tax Expense 161,000

90. A tax imposed in the nature of a penalty to the corporation to


deter tax avoidance of shareholders who avoid paying the dividends
tax on the earnings distributed to them by the corporation.
a. Minimum corporate income tax
b. Optional corporate income tax
c. Improperly accumulated earnings tax
d. Capital gains tax

91. First statement: An accumulation of earnings or profits (including


undistributed earnings or profits of prior years) is unreasonable
if it is not necessary for the purpose of the business,
considering all the circumstances of the case.
Second statement: The term "reasonable needs of the business" are
hereby construed to mean the immediate needs of the business,
including reasonably anticipated needs.
a. Only the first statement is correct
b. Only the second statement is correct
c. Both statements are correct
d. Both statements are incorrect

92. The Improperly Accumulated Earnings Tax (IAET) is imposed on


improperly accumulated taxable income earned starting January 1,
1998 by domestic corporations as defined under the Tax Code and
which are classified as closely-held corporations at the rate of:
a. twenty percent (20%). c. ten percent (10%).
b. fifteen percent (15%). d. five percent (5%).

93. The Improperly Accumulated Earnings Tax shall not apply to which
of the following corporations?
a. Banks and other non-bank financial intermediaries
b. Insurance companies
c. Publicly-held corporation
d. All of the choices

94. First statement: Once the profits have been subjected to


improperly accumulated earnings tax, the same shall no longer be
subject to the same tax in later years even if not declared as
dividends.
Second statement: Profits which have been subjected to improperly
accumulated earnings tax when finally declared as dividends shall
be subject to tax on dividends.
a. Both statements are true
b. Both statements are false
c. Only the first statement is true
d. Only the second statement is true

Downloaded by jaene gallora (nichjaene05@gmail.com)


lOMoARcPSD|5612816

95. To avoid payment of IAET, when must the dividends be declared and
paid or issued?
a. Not later than one year following the close of the taxable
year
b. Not later than the 15th day following the close of the
taxable year
c. Not later than 60th day following the close of the taxable
quarter
d. None of the choices

TAX REMEDIES
96. Rosalie, a compensation income earner, filed her income tax return
for the taxable year 2013 on March 30, 2014. On May 20, 2017,
Rosalie received an assessment notice and letter of demand
covering the taxable year 2013 but the postmark on the envelope
shows April 10, 2017. Her return is not a false and fraudulent
return. Can Rosalie raise the defense of prescription?
a. No. The 3 year prescriptive period started to run on April
15, 2014, hence, it has not yet expired on April 10, 2017.
b. Yes. The 3 year prescriptive period started to run on April
15, 2014, hence, it had already expired by May 20, 2017.
c. No. The prescriptive period started to run on March 30,
2014, hence, the 3 year period expired on April 10, 2017.
d. Yes. Since the 3-year prescriptive period started to run on
March 30, 2014, it already expired by May 20, 2017.

97. On March 30, 2012 Emmett Foods, Inc. received a notice of


assessment and a letter of demand on its April 15, 2009 final
adjustment return from the BIR. Emmett Foods then filed a request
for reinvestigation together with the requisite supporting
documents on April 25, 2012. On June 2, 2012, the BIR issued a
final assessment reducing the amount of the tax demanded. Emmett
Foods was satisfied with the reduction, it did not do anything
anymore. On April 15, 2017 the BIR garnished the corporation's
bank deposits to answer for the tax liability. Was the BIR action
proper?
a. Yes. The BIR has 5 years from the filing of the protest
within which to collect.
b. Yes. The BIR has 5 years from the issuance of the final
assessment within which to collect.
c. No. The taxpayer did not apply for a compromise.
d. No. Without the taxpayer’s prior authority, the BIR action
violated the Bank Deposit Secrecy Law.

98. Renesmee, Inc. received a notice of assessment and a letter from


the BIR demanding the payment of P3 million pesos in deficiency
income taxes for the taxable year 2015. The financial statements
of the company show that it has been suffering financial reverses
from the year 2016 up to the present. Its asset position shows
that it could pay only P500,000.00 which it offered as a
compromise to the BIR. Which among the following may the BIR

Downloaded by jaene gallora (nichjaene05@gmail.com)


lOMoARcPSD|5612816

require to enable it to enter into a compromise with Renesmee,


Inc.?
a. Renesmee must show it has faithfully paid taxes before 2016.
b. Renesmee must promise to pay its deficiency when financially
able.
c. Renesmee must waive its right to the secrecy of its bank
deposits.
d. Renesmee must immediately deposit the P500,000 with the BIR.

99. As a rule, within what period must a taxpayer elevate to the Court
of Tax Appeals a denial of his application for refund of income
tax overpayment?
a. Within 30 days from receipt of the Commissioner’s denial of
his application for refund.
b. Within 30 days from receipt of the denial which must not
exceed 2 years from payment of income tax.
c. Within 2 years from payment of the income taxes sought to be
refunded.
d. Within 30 days from receipt of the denial or within two
years from payment.

100. What is the effect on the tax liability of a taxpayer who does not
protest an assessment for deficiency taxes?

a. The taxpayer may appeal his liability to the CTA since the
assessment is a final decision of the Commissioner on the
matter.
b. The BIR could already enforce the collection of the
taxpayer's liability if it could secure authority from the
CTA.
c. The taxpayer's liability becomes fixed and subject to
collection as the assessment becomes final and collectible.
d. The taxpayer's liability remains suspended for 180 days from
the expiration of the period to protest.

101. The taxpayer seasonably filed his protest together with all the
supporting documents. It is already July 31, 2017, or 180 days
from submission of the protest but the BIR Commissioner has not
yet decided his protest. Desirous of an early resolution of his
protested assessment, the taxpayer should file his appeal to the
Court of Tax Appeals not later than
a. August 31, 2017.
b. August 30, 2017.
c. August 15, 2017.
d. August 1, 2017.

Downloaded by jaene gallora (nichjaene05@gmail.com)


lOMoARcPSD|5612816

DOCUMENTARY STAMP TAX


102. A newly formed corporation issued shares of stocks to its
incorporators for P150,000. The par value of the shares issued is
P100,000. How much is the documentary stamp tax?
a. P500 c. P750
b. P1,000 d. P1,500

Par Value Divide by 200 x 1 peso = 100,000 / 200 = 500

103. Mr. T invested in shares of stock of Kapisananngmga Sisters Inc.


amounting to P100,000 with par value of P80,000. After 2 years, he
disposed said shares directly to Mr. B for P230,000.
The documentary stamp tax on above transaction is:
a. P400 c. P300
b. P1,150 d. P862.50

**80,000 / 200 = 400 x 75% = 300

104. Based on number 97, but assuming the shares are without par value,
how much is the documentary stamp tax?
a. P500 c. P750
b. P1,000 d. P0.00

**Selling Price - Cost = 230,000 - 80,000 = 150,000 / 200 = 750

105. Continuing number 99, if the shares were subsequently sold for
P200,000, how much is the documentary stamp tax?
a. P750 c. P1,500
b. P187.50 d. P375

**Doc Stamp Upon Original Issuance 750 x 25% = 187.50

106. Mr. Purisima owns a resthouse in Pampanga acquired by him for


P10,750,000. He sold the same to Mr. Apo for P5,000,000. The fair
market value at the time of sale per assessor’s office is
P10,000,000 while zonal value is P15,000,000. The documentary
stamp tax on the transaction is:
a. P75,000 c. P150,000
b. P225,000 d. P161,250

**15,000,000/1000 = 15,000 x 15 pesos = 225,000

107. Who is liable to the payment documentary stamp tax?


a. Mr. Purisima
b. Mr. Apo
c. It depends on the agreement of the parties
d. It depends on who is benefiting on the transaction

Downloaded by jaene gallora (nichjaene05@gmail.com)


lOMoARcPSD|5612816

TAXATION QUIZZER PART 2

1. The BIR is assessing deficiency withholding tax on total Repairs and


Maintenance claimed by non-stock non-profit organization. The BIR
alleges that foundation failed to withhold the correct amount of the
2% expanded withholding tax due on the Repairs and
Maintenance expense. As a CPA assisting the foundation in the BIR tax
investigation, you can reason that ___.
a. The BIR should have considered the 1% expanded withholding tax
instead of the 2%in determining the deficiency tax.
b. The foundation is exempt from the tax and hence is not liable to
the deficiency withholding tax.
c. The foundation is exempt from tax and hence is not required to
withhold tax on its expenses.
d. The BIR should have considered both the 1% and 2% withholding tax
in determining the deficiency tax.

2. Which of the following statement is true?


a. Taxes are in the nature if contracts between the taxpayers and
the government.
b. Taxes and debts are similar nature and character.
c. As the general rule, no set-off is allowed against the demands
for taxes levied for general or local government purposes.
d. In taxation, the personal consent of the individual taxpayers is
required.

3. Which of the following is not an attribute of tax?


a. It is an enforced contribution on a person taxed.
b. It is levied by the executive department of the state.
c. It is imposed by the state which has jurisdiction over the
person, property, or excises.
d. It is generally in money

4. Which of the statement below is grammatically correct?


a. The imposition of minimum corporate income tax may be suspended
if substantial losses are sustained due to a prolonged labor
dispute.
b. The imposition of minimum corporate income tax maybe suspend if
substantial losses are sustained due to prolonged labor dispute.
c. The imposition of minimum corporate income tax may be suspended
if substantial losses are sustained due to a prolong disputes.
d. The imposition of minimum corporate income tax may be suspended.
If substantial loss are sustained due to a prolonged labor
dispute.

5. Which of the following statements is correct?


a. If what is delegated is tax administration, the delegation is
invalid.
b. If what is delegated is tax administration, the delegation is
valid.
c. Tariff powers cannot be delegated to the President.
d. As a general rule, taxation can be further delegated.

Downloaded by jaene gallora (nichjaene05@gmail.com)


lOMoARcPSD|5612816

6. The BIR issued a tax assessment against the taxpayer who was not given
sufficient time to protest the said assessment. The taxpayer noted
that their competitors were issued tax assessments but were given
enough time to protest. The BIR violated the _____________ of the
constitution.
a. Equal protection clause c. Equitability principle
b. Due process clause d. Uniformity principle

Items 7-10 are based on the following information:

On March 30, 2012, XXX, Inc., received a notice of assessment and a letter
of demand on its April 15, 2009 final adjustment return from the BIR.
XXX, Inc., then filed a request for reinvestigation together with the
requisite supporting documents on April 25, 2012. On June 2, 2012, the
BIR issued a final assessment reducing the amount of the tax demand.
Since XXX, Inc., was satisfied with the reduction, it did not do
anything anymore. On April 15, 2017 the BIR garnished the
corporation’s bank deposits to answer the liability.

7. Was the BIR action proper?


a. No, the taxpayer did not apply for the compromise.
b. Yes, the BIR has 5years from the filing within which to collect.
c. No, without the taxpayer’s prior authority, the BIR action
violated the Bank Deposit Secrecy Law.
d. Yes, the BIR has 5 years from the issuance if the final
assessment within which to collect.

8. What is the effect of the XXX, Inc.’s failure to file a protest on its
assessed deficiency taxes?
a. The taxpayer may file a motion for reconsideration to the CIR on
the matter.
b. The taxpayer may appeal his liability to the CTA since the
assessment is a final decision of the Commissioner on the matter.
c. The taxpayer’s liability becomes fixed and subject to collection
as the assessment becomes final and collectible.
d. The BIR could already enforced the collection of the taxpayer’s
liability if it could secure authority from the CTA.

9. Which statement is correct? The collection of a deficient tax


assessment by distraint and levy:
a. May be done only once during the taxable year.
b. Must be done successively, first by distrait and then by levy;
c. May be repeated, if necessary, until the full amount due,
including all expenses, is collected;
d. Automatically covers the bank deposits of a delinquent taxpayer.

10. Which of the following is grammatically correct?


a. Tax assessment refer to the process of determining the correct
amount of tax due in accordance with the prevailing tax laws.
b. Tax assessment refer to the process of determining the correct
amount of tax due in accordance with the prevailed tax laws.

Downloaded by jaene gallora (nichjaene05@gmail.com)


lOMoARcPSD|5612816

c. Tax assessment refers to the process of determining the correct


amount of tax due in accordance with the prevailed tax laws.
d. Tax assessment refers to the process of determining the correct
amount of tax due in accordance with the prevailing tax laws.

11. A joker was commissioned in a kiddy party to perform magic. The comic
was to be paid P100, 000 for his performance and the parties signed
the necessary contract. He then gratuitously assigned his rights under
the contract to his son. The son later on collected the P100,
000 talent fee of his father from the contractee. The national
internal revenue tax/es payable is/are:
a. Income tax only.
b. Donor’s tax only.
c. Both income and donor’s taxes.
d. Neither income tax nor donor’s tax.

12. Which one of the following statements is wrong?


a. Income out of the labor of the wife is conjugal property.
b. Income out of the separate property of the husband is conjugal
property.
c. Amount receivable as retirement benefit under R.A. No. 4917
during the marriage is conjugal property.
d. Property received is donation when the fair value was P2,
000,000, resulting in a gain of P500, 000. The gain is conjugal
property.

13. Amount receivable by the estate of the deceased, his executor or


administration as a beneficiary under life insurance policy taken by
the decedent upon his own life is:
a. Excluded from gross state.
b. Part of gross state if the beneficiary is revocable.
c. Excluded from gross state if the beneficiary is irrevocable.
d. Part of gross state whether the beneficiary is revocable or
irrevocable.

14. The following journal entry was made in the purchases journal of a
VAT-registered taxpayer:

Purchases xxx
Cash or Accounts Payable xxx

The journal entry signifies that:


a. Purchases were from a non-VAT supplier.
b. Purchases were from a VAT-registered supplier.
c. Purchases were either from non-VAT or VAT-registered supplier.
d. Input tax has been taken as part of the cost of purchase.

15. A common carrier by land is engaged in the transport of passengers,


goods and cargoes. He is not VAT-registered. What business tax or
taxes is he liable to pay?
a. 12% value-added tax
b. 3% common carrier’s tax

Downloaded by jaene gallora (nichjaene05@gmail.com)


lOMoARcPSD|5612816

c. 12% VAT on gross receipts from transport of goods and cargoes and
3% common carrier’s tax on gross receipts from transport of
passengers.
d. 3% tax on VAT-exempt persons on gross receipts from transport of
passengers.

16. Statement 1. As to the property of the state, exemption is the rule


and taxation the exception.
Statement 2. As of the property of the taxpayer, taxation is the rule and
exemption the exception.

a. Both statements are correct


b. Both statements are incorrect.
c. Statement 1 is correct while statement 2 is incorrect.
d. Statement 1 is incorrect while statement 2 is correct.

17. Foreign income taxes paid by the resident citizen or domestic


corporation.
a. May be claimed only as tax credit.
b. May be claimed only as tax deduction.
c. Do not qualify either as a tax credit or as a tax deduction.
d. May be claimed either as a tax credit or as a tax deduction at
the option of the income taxpayer.

18. Statement 1. Government agencies performing essential governmental


functions are subject to tax unless expressly exempted.
Statement 2. Government agencies performing propriety function are exempt
from tax unless expressly taxed.

a. Both statements are correct.


b. Both statements are incorrect.
c. Statement 1 is true while statement 2 is false.
d. Statement 2 is true while statement 1 is false.

19. Statement 1. Tax avoidance or tax minimization is the use by the


taxpayer of legally permissible methods in order to reduce tax
liability.
Statement 2. Tax evasion or tax dodging is the use by the taxpayer of
illegal means to defeat or lessen the payment of tax.

a. Both statements are correct.


b. Both statements are incorrect.
c. Statement 1 is correct while statement 2 is incorrect.
d. Statement 2 is correct while statement 1 is incorrect.

20. Which one among the following items below is included in the gross
state?
a. Revocable transfer.
b. Transfer with reservation of certain rights.
c. Transfer under general power of appointment.
d. Transfer in contemplation of death which is onerous.

Downloaded by jaene gallora (nichjaene05@gmail.com)


lOMoARcPSD|5612816

21. Which of the following statement is correct?


a. The final tax on compensation of special kind of non-resident
aliens is 25% of the gross income.
b. Interest income from a foreign currency deposit unit in the
Philippines of a non resident alien is not subject to final tax.
c. Informer’s reward is subject to final tax of 10% based on the 10%
of the value of tax assessed or P1, 000,000 whichever is higher.
d. Prizes exceeding P10,000 derived by non-resident alien not engage
in trade or business here in the Philippines is subject to a
final tax of 20%

22. The power to decide disputed assignment, refunds of internal revenue


taxes, fees or other charges, penalties imposed in relation thereof,
or other matters arising other the tax code or other laws or portions
thereof administered by the BIR is vested in the:
a. Secretary of Finance
b. Commissioner of Internal Revenue
c. Court of Tax Appeals
d. Regional Trial Court

23. The commissioner of the Internal Revenue (CIR) is prohibited by law to


look into the bank accounts of taxpayer, except when:
a. Taxpayer is accused of heinous crime
b. Taxpayer did not invoke his right to privacy during the tax
audit.
c. Taxpayer applies for compromise on tax obligation on account of
financial incapacity.
d. The CIR has reason to believe that taxpayer has filed a false or
fraudulent return.

24. A VAT-registered realty company sells real property in the course of


its business. On April 30, 2016, it has sold a lot under the following
items (VAT excluded).

Selling Price P4,000,000


Down payment, 4/30/16 400,000
1st Installment, 4/30/17 600,000
2nd Installment, 4/30/18 1,000,000
3rd Installment, 4/30/19 1,000,000
4th Installment, 4/30/20 1,000,000
Interest and other charges of 10%
On unpaid balance per installment

The zonal value of the lot at the time of sale is P4, 800,000.

The output VAT for the installment received on April 30, 2017, is:
a. P43,200
b. P86,400
c. P115,200
d. P129,600

Downloaded by jaene gallora (nichjaene05@gmail.com)


lOMoARcPSD|5612816

Output Tax (4,800,000 x 12%) = 576,000

Principal 600,000 / 4,000,000 x 576,000 = 86,400


Interest 3,600, 000 x 10% x 12% = 43,200
Total 129,600

25. Counting No. 24, the output VAT on April 30, 2020, is:
a. P132, 000
b. P144,000
c. P156,000
d. P228,000

4,800,000 x 12% = 576,000 out put tax


Principal 1,000,000 / 4,000,000 x 576,000 = 144,000
Interest 1,000,000 x 10% x 12% = 12,000
Total 156,000

26. Assuming that the real property in No. 24 is the residential lot sold
for cash of P1, 750,000 (VAT not separately blend in the sales
document) on April 30, 2016. Zonal value of the lot at the time of
sale is P1, 900,000. The output VAT on the sale is:
a. P187,500
b. P210, 000
c. P228,000
d. None, as it is exempt the VAT.

27. The lease of real or personal property is subject to:


i. 7% gross receipt tax (GRT) if the lessor is the bank.
ii. 3% percentage tax or 12% value-added tax (VAT) if the
lessor is not a bank.

a. No to I and II
b. Yes to I and II
c. Yes to I only
d. Yes to II only

28. Ayala Land, Inc. (ALI) bought a parcel of land in 2014 for P7 million
as part of its inventory of real properties. In 2016, it sold the land
for P12 million which was its zonal valuation. In the same year, it
suffered a loss of P6 million for selling another parcel of land from
its inventory. These were the only transactions ALI had in its real
estate business. Which of the following is the applicable tax
treatment?
a. ALI shall be subject to a tax of 6% of P12 million.
b. ALI’s gain of P5 million shall be subject to holding period.
c. ALI could deduct its P6 million loss from its P5 million gain.
d. ALI’s P6 million loss could not be deducted from its P5 million
gain.

29. This is an inherent limitation on the power of taxation:


a. The rule of taxation shall be uniform and equitable.
b. No law impairing the obligations of contacts shall be enacted.

Downloaded by jaene gallora (nichjaene05@gmail.com)


lOMoARcPSD|5612816

c. Tax laws can not apply to the property of foreign governments.


d. Charitable institutions, churches, parsonages, convents and all
lands, buildings and improvements, actually, directly, and
exclusively used for religious, charitable or educational purposes
shall be exempt for taxation.

30. X took a life insurance policy of P5 million where the monthly


premium is P10, 000. The proceeds will be paid to X after 25 years
to the X’s estate should X die before completing the equivalent of
25 years payment. If the X outlived the policy, which of the
following is correct?
a. The proceeds will be part of X’s gross estate.
b. The proceeds will be part of X’s taxable income.
c. The proceeds will be party taxable estate and partly exempt.
d. The proceeds will be party taxable income and partly exempt.

31. Containing the preceding number, except that after paying the
equivalent of ten years premium, X transferred the policy to Y for
P1.5 million and Y continued paying the monthly premium as they
mature. After 10 years, X died. Which of the following is correct?
a. The proceeds will be part of X’s gross estate.
b. The proceeds received by Y is part of his taxable income.
c. The amount received by X from Y is part of X’s taxable income.
d. The amount received by the X from Y and the proceeds received by Y
are partly taxable income and partly exempt.

32. An owner of several warehouses for rent, which used to be VAT-


exempt because its annual gross receipts never exceeded P1,919,500
decided to register under the VAT system on January 2, 2016. The
following data were from the first quarter ending March 31, 2016:

Rental from warehousing services, net of VAT P 672,000


Purchases of supplies, gross of VAT 224,000
Inventory of supplies, January 1, 2016 201,600
Actual VAT paid on the inventory of
supplies, January 1, 2016 21,600

The value added tax payable for the quarter is:


a. P23,520
b. P35,040
c. P41,088
d. P52,608

VAT on rental (672,000 x 12 %) 80,640


VAT on purchase of supplies (224,000 x 12/112) (24,000)
Transitional Input Tax(higher) (21,600)
VAT Payable 35,040

Downloaded by jaene gallora (nichjaene05@gmail.com)


lOMoARcPSD|5612816

33. Assuming that one vacant warehouse in the preceding number was
rented out for the whole month of April 2016 and received P107, 000
as rent, gross of VAT but net of the applicable creditable
withholding income tax (expanded). The output VAT on the rental
receipt is:
a. P12,000
b. P12,240
c. P12,840
d. P13,440

107,000/107% = 100,000 x 12% = 12,000

Numbers 34 to 42 are based on the following information:

Dina Cabangon, a citizen and resident of the Philippines, died on November


1, 2016. Her marriage was under the system of absolute community of
property. The following properties and obligations were left:

Property received by Dina as inheritance on February 1, 2015


(during the marriage) P2,000,000
Real property acquired through the labor of both Dina and
her Husband during the marriage (family home) 4,000,000
Property owned by Dina before marriage 300,000
Property owned by Dina’s Husband before marriage 200,000
Funeral expenses 300,000
Unpaid mortgage on property inherited 200,000
Judicial expenses for the settlement of the estate 80,000
Unpaid obligations (excluding the unpaid mortgage) 40,000

The property received as inheritance was part of the gross estate of the
prior decedent at a fair market value of P1,100,000. At the time of
inheritance, it was mortgage for P300,000. Dina was able to pay P100,000
before she passed away.

34. The total community property is:


a. P6,500,000 c. P4,000,000
b. P4,500,000 d. P2,300,000

35. The total exclusive property is:


a. P4,200,000 c. P2,300,000
b. P2,500,000 d. P2,000,000

36. The total ordinary community deduction is:


a. P200,000 c. P420,000
b. P320,000 d. P445,000

37. The total ordinary exclusive deduction (excluding vanishing


deduction) is:
a. P400,000 c. P200,000
b. P300,000 d. P100,000

Downloaded by jaene gallora (nichjaene05@gmail.com)


lOMoARcPSD|5612816

38. The deduction for family home is:


a. P0 c. P2,000,000
b. P1,000,000 d. P4,000,000

39. The amount of vanishing deduction is:


a. P0 c. P736,000
b. P816,000 d. P656,000

40. The total special deduction is:


a. P0 c.P2,000,000
b. P1,000,000 d.P4,000,000

41. The taxable net estate is:


a. P1,154,000 c. P3,154,000
b. P2,154,000 d. P4,244,000

SOLUTION:
Separate Common Total
2,000,000 4,000,000
300,000
200,000
Gross Estate 2,000,000 4,500,000 6,500,000
Funeral Expenses:
Actual 300,000
Limit (6.5 x 5%) 325,000
Threshold 200,000
Whichever is lower (200,000)
Judicial Expenses (80,000)
Indebtedness (200,000) (40,000)
Vanishing Deductions* (736,000)
Net Estate After
Ordinary Deductions 1,064,000 4,180,000 5,244,000
Family Home (1,000,000)
Standard Deductions (1,000,000)
Share of Surviving Spouse 1/2 x 4,180,000 (2,090,000)
Taxable NE 1,154,000

*Vanishing Deduction:
Lower FMV 1,100,000
Mortgage Assumed and Paid (100,000)
Initial Basis 1,000,000
Pro Rated Deductions:
1,000,000/6,500,000 x 520,000 (80,000)
Final Basis 920,000
Vanishing Rate _ 80%__
Vanishing Deductions 736,000

42. Going back to the original problem, except that the marriage of
Dina Cabangon to her spouse was under the system of conjugal
partnership of gains. The taxable net estate would be:
a. P3,206,032 c. P1,206,032
b. P2,206,033 d. P1,154,000

Downloaded by jaene gallora (nichjaene05@gmail.com)


lOMoARcPSD|5612816

Separate Common Total


2,000,000 4,000,000
300,000
Gross Estate 2,300,000 4,000,000 6,300,000
Funeral Expenses:
Actual 300,000
Limit (6.5M x 5%) 325,000
Threshold 200,000
Whichever is lower (200,000)
Judicial Expenses (80,000)
Indebtedness (200,000) (40,000)
Vanishing Deductions** (733,968)
Net Estate After
Ordinary Deductions 1,366,032 3,680,000 5,046,032
Family Home (1,000,000)
Standard Deductions (1,000,000)
Share of Surviving Spouse 1/2 x 4,180,000 (1,840,000)
Taxable NE 1,206,302

**Vanishing Deduction:
Lower FMV 1,100,000
Mortgage Assumed and Paid (100,000)
Initial Basis 1,000,000
Pro Rated Deductions:
1,000,000/6,300,000 x 520,000 (82,540)
Final Basis 917,460
Vanishing Rate 80%
Vanishing Deductions 733,968

43. In March 2016, Imelda, who is fond of jewelries, bough the


following: diamond ring for P750, 000; bracelet for P250, 000;
necklace for P500, 000; and a brooch for P500, 000. Imelda drives
income from the exercise of her profession as a topnotch Interior
Designer. In October 2016. Imelda sold her diamond ring, bracelet
and necklace for only P1.25 million, incurring a loss of P250, 000.
She used the P1.25 million to buy a solo diamond ring in November
2016 which she sold for P1.5 million in September 2017. Imelda had
no other transaction on jewelry in 2017. Which among the following
best describes the tax implications arising from the aforesaid
transactions?

a. Imelda may carry over and deduct her 2016 loss only from her 2017
gain.
b. Imelda may deduct her 2016 loss from both her 2017 professional
income.
c. Imelda may not deduct her 2016 loss from both her 2017 professional
income and her gain.
d. Imelda may carry over and deduct her 2016 loss from her 2017
professional income as well as from her gain.

Downloaded by jaene gallora (nichjaene05@gmail.com)


lOMoARcPSD|5612816

44. Taxation could be exercise by the following except one. Which one?
a. Judiciary
b. Legislative
c. Local government unit
d. President of the Philippines, in certain cases.

45. Stages, aspects or processes in taxation.


a. Levy of the tax
b. Collection of the tax.
c. Payment of the tax by the taxpayer.
d. All of the above.

46. Statement 1. Onerous donations are subject to donor’s tax.


Statement 2. Gratuitous donations are not subject to donor’s tax.
a. Both statement s are true
b. Both statements are false
c. Only statement 1 is true but not statement 2.
d. Only statement 2 is true but not statement 1.

47. Which of the following statements are correct?


a. Gift-splitting is a form of tax dodging.
b. The uncle who is the brother of the donor’s mother-in-law is a
non-stranger to the donor for purposes of the donor’s tax.
c. A gift made to a relative in January 2016 is to be added to the
gift made to the same relative in December 2015 in determining
the gift tax.
d. Renunciation by an heir including the surviving spouse of his/her
share in the hereditary estate left by the decedent is subject to
donor’s tax if done in favor of identified heirs to the exclusion
or disadvantage of the other co-heir/s in the hereditary estate.

48. Mistah, single and sales executive of a leading pharmaceutical firm


(RiteMed), received in 2016 the following from his employer:

Salary, net of P267,000 withholding tax P 683,000


Allowances and benefits received:

 Rent paid by RiteMed on the house which Mistah occupies for


residential purposes, net of 5% withholding 129,200

 Entertainment allowance subject to liquidation (P75,000 was duly


receipted in the name of RiteMed and used to entertain RiteMed’s
customers and the balance of P25,000 was used to purchase a late
model mobile phone for the personal use of Mistah) 100,000

 Reimbursement of entertainment expenses paid by Mistah (P17,500


was used to entertain Mistah’s boyhood pals and the balance of
P22,500 was used to promote RiteMed’s businesses.)
40,000

Downloaded by jaene gallora (nichjaene05@gmail.com)


lOMoARcPSD|5612816

 Fixed yearly allowance for entertainment 85,000

The fringe benefit tax is:


a. P50,400
b. P52,000
c. P84,000
d. P92,000

Gross Rent on Housing 129,200/95% = 136,000 x 50% = 68,000


Expense Account (CP & Pal Exp) 25,000 + 17500 42,500
Total 110,500

110,500/68% 162,500
162,500 x 32% 52,000

49. Continuing number 48, the income tax payable by Mistah is:
a. P13,200 old law
b. P27,760
c. P29,200
d. P43,360

Compensation(683,000 + 267,000) 950,000


Fixed Year Allowance 85,000
Personal Exemptions (50,000)
Taxable Income 985,000

Income Tax per Tabular (OLD LAW)


up to 500,000 125,000
Excess (485,000 x 32%) 155,200
Withholding Tax (267,000)
Income Tax Payable 13,200

Income Tax per Tabular (TRAIN LAW)


up to 800,000 130,000
Excess (185,000 x 30%) 55,500
Withholding Tax (267,000)
Income Tax Payable (Refund) (81,500)

Numbers 50 to 56 are based on the following information:


Domestic Export
Sales to private entities P500,000 P500,000
Sales to the government 500,000
Sales of exempt goods 500,000

Input taxes passed on by VAT-registered suppliers on:


Sales to private entities 30,000 20,000
Sales to the government 25,000
Sales of exempt goods 10,000
Purchase of depreciable capital goods not
attributable to any specific activity (monthly
amortization for 60 months) 120,000

Downloaded by jaene gallora (nichjaene05@gmail.com)


lOMoARcPSD|5612816

The sales to the government were subjected to the automatic deduction of the
1% creditable withholding tax (CWT) on its purchases from domestic
suppliers.

50. The value-added tax payable on the domestic sales to private


entities is:
a. Zero
b. (P10,000)
c. P60,000
d. P70,000

51. The total input taxes attributable to zero-rated sales is:


a. P20,000
b. P30,000
c. P50, 000
d. P60, 000

52. If the input taxes attributable to zero-rated are claimed as tax


credit, the net input value-added tax refundable is:
a. Zero
b. P40,000
c. P50,000
d. P60,000

53. The actual input taxes attributable to the domestic sales to the
government is:
a. P25,000
b. P30,000
c. P55,000
d. P65,000

54. The value-added tax payable on the domestic sales to the government
which was withheld as final withholding VAT is:
a. P5,000
b. P25,000
c. P30,000
d. P35,000

55. The journal entry to take up the domestic sales to the government
is:
a. Cash/AR 560,000
Sales 560,000
b. Cash/AR 560,000
Sales 500,000
Output tax 60,000
c. Cash/AR 535,000
Final withholding VAT 25,000
Sales 500,000
Output tax 60,000

Downloaded by jaene gallora (nichjaene05@gmail.com)


lOMoARcPSD|5612816

d. Cash/AR 530,000
Final withholding VAT 25,000
Creditable withholding tax 5,000
Sales 500,000
Output tax 60,000

Private Entities
VAT 0% VAT Government
OT 500,000 x 12% 60,000 60,000
500,000 x 0% -0-
IT (30,000) (20,000)
500,000 x 7% (35,000)

120,000 x 1 / 4 (30,000) (30,000)


VAT Payable -0-
Excess Input Tax (50,000)
FINAL VAT Withheld 25,000

56. The journal entry to reflect the excess of actual input VAT-over
the statutorily allowed input tax on the domestic sales to the
government is:
a. Output tax 60,000
Revenue and expense summary 20,000
Input tax 55,000
Final withholding VAT 25,000
b. Output tax 60,000
Input tax 15,000
Final withholding VAT 25,000
Revenue and expense summary 20,000
c. Output tax 60,000
Input tax 55,000
Revenue and expense summary 5,000
d. Output tax 60,000
Revenue and expense summary 20,000
Input tax 55,000
Cash/AP 25,000

57. Which one among the following statements is wrong?


a. Private Banks may be authorized to collect internal revenue taxes.
b. The Bureau of Customs (BOC) is also charged with the collection of
internal revenue taxes.
c. The local government unit’s (LGUs), such as cities, Municipalities
and Provinces, from part of the national tax system.
d. The bureau of Internal Revenue (BIR) is part of the administrative
machinery for the assessment and collection of internal revenue
taxes.

58. The prescriptive period for the issuance of a formal letter of


demand and final assessment notice (FLD/FAN) may not ordinarily be
stayed because of the lifeblood theory. There are certain
instances, however, where the running of the prescriptive period

Downloaded by jaene gallora (nichjaene05@gmail.com)


lOMoARcPSD|5612816

may be suspended. Which among the following instances is not among


the recognized exceptions which suspend the prescriptive period
within which to assess?
a. If the taxpayer is out of the country.
b. If the taxpayer changes his address, informing the commissioner of
such change.
c. Where the taxpayer request for and is granted a re investigation by
the Commissioner.
d. When the Commissioner of Internal Revenue (CIR) is prevented from
making an assessment and within 60 days thereafter.

59. An examination of a calendar year corporate taxpayer’s records


shows that it filed its final adjustment income tax return on
February 15, 2016 for its 2015 income. It subsequently filed an
amended income tax return March 21, 2016. Up to what date is the
Bureau of Internal Revenue (BIR) within which to issue a formal
letter of demand and final assessment notice (FLD/FAN)?
a. December 31, 2018 c. March 21, 2019
b. February 15, 2019 d. April 15, 2019

60. A closely held corporation has initially offered its shares in the
Philippine stock exchange (PSE). The following data pertain to the
initial public offering (IPO):

Number of shares sold in IPO 1,000,000 shares


Total outstanding shares before
the listing in the PSE 3,000,000 shares
Gross value in money of the IPO P20,000,000
The percentage tax due is:
a. P100,000 c. P400,000
b. P200,000 d. P800,000

Gross Value of IPO 20,000,000


OPT Rate 1M / 4M = 25% ( 4%, from 4%,2%, 1%) 4%
Percentage Tax 800,000

61. Shares of stock in a domestic corporation held as investment when


sold not through the local stock exchange shall be subject to:
a. 3% OPT or 12% VAT based on gross income
b. ½ of 1% based on gross selling price or gross value in money.
c. 5% on first P100, 000 capital gain; 10% on excess of P100, 000.
d. 4%;2%;1% based on gross selling price or gross value in money.

62. Malakas and Maganda were legally separated. They have six minor
children, all qualified to be claimed as additional exemptions for
income tax purposes. The court awarded custody of two of the
children to Malakas and three to Maganda, with Malakas directed
provide full financial support for them as well. The court awarded
the sixth child to Malakas’ parents with Malakas also providing
full financial support. Assuming that only Malakas is
gainfully earning while Maganda is not, for how many children could

Downloaded by jaene gallora (nichjaene05@gmail.com)


lOMoARcPSD|5612816

Malakas claim as additional exemptions when he files his annual


income tax return?
a. Two children c. Five Children
b. Three children d. Six children

63. Statement 1. Health and/or hospitalization insurance premium is


deductible from gross income by the spouse who claimed the
additional exemptions in case of married income taxpayers.
Statement 2. Health and/or hospitalization insurance premium paid
by an individual income taxpayer is deductible from gross income
for a minimum amount of P2, 400 provided the family’s gross income
for the year does not exceed P250, 000.
a. Both statements are true
b. Both statements are false
c. Statement 1 is true while statement 2 is false.
d. Statement 2 is true while statement 1 is false.

64. The following fringe benefits were given by an employer to its


employees for the quarter ending September 30, 2016:

De minimis benefits (not exceeding the maximum) P 200,000


Reimbursed expense of rank and file employees 400,000
Housing benefits to managers and supervisors
(Representing total rents) 680,000

The fringe benefit tax payable for the quarter is:


a. P160, 000 c. P442, 353
b. P320, 000 d. P502, 353

Housing Benefits to Manager


680,000 x 50% = 340,000 / 68% x 32% = 160,000

65. Continuing number 64, how much would be the total deductions from
gross income which may be claimed by the employer?
a. P1,100,000 c. P1,440,000
b. P1,280,000 d. P1,600,000

De minimis benefits (not exceeding the maximum) P 200,000


Reimbursed expense of rank and file employees 400,000
Housing benefits to managers and supervisors
(Representing total rents) 680,000
Fringe Benefit Tax Expense 160,000
Total Deductions from Gross Income 1,440,000

66. Going back to the number 64, the employer shall file a remittance
return of the final tax on fringe benefit and pay the tax withheld
within:
a. 5 days from the close of each month.
b. 10 days from the close of each month.
c. 10 days from the close of each fiscal quarter.
d. 10 days from the close of each calendar quarter.

Downloaded by jaene gallora (nichjaene05@gmail.com)


lOMoARcPSD|5612816

67. Kalansay, a native of Negros, died leaving a property acquired by


purchase from Naty Gok who died 3 ½ years ago. The property is now
a Kalansay’s gross estate. The estate’s vanishing deduction rate
is:
a. 0%
b. 20%
c. 40%
d. 60%
Property is not gratuitously acquired by the present Estate

68. A lessor of residential units has the following gross receipts for
2016:
Monthly Number
Rent per unit of units Total
P 9,000 5 P 540,000
10,000 5 600,000
11,000 5 660,000
12,000 5 720,000

The output VAT is:


a. P0 c. P270,000
b. P230,340 d. P302,400
Leasing on a residential units is VAT Exempt

69. Using the same facts in number 68, only that the lessor is a VAT-
registered person, the OPT tax due is:
a. P0 c. P67,500
b. P57,585 d. P75,600
OPT Exempt also

70. The deduction allowed for the payment of premium on hospitalization


insurance during the taxable year by a resident citizen amounting
to P3, 000 for the months of August to December is:
a. P 3,000
b. P2, 400
c. P1, 200
d. P1, 000
200 x 5 months = 1,000

71. The following are among the constitutional limitation on the power
of taxation. Which one is not?
a. Due process clause
b. Non-impairment clause
c. Equal protection clause
d. No imprisonment for non-payment of tax

Downloaded by jaene gallora (nichjaene05@gmail.com)


lOMoARcPSD|5612816

72. Pacmom filed her income tax from return for 2015 on May 16, 2016
and paid the tax of P50,000. Upon audit by the BIR an assessment
notice was issued on April 30, 2019, requiring Pacmom to pay a
deficiency tax of P 75,000 not later than July 30, 2019. Pacmom
will:
a. False prescription as defense
b. Request for an extension of time to pay the deficiency income tax.
c. Go to the court of tax appeals to appeal the assessment made by the
BIR.
d. Ignore the assessment as the date of collection is already way
beyond three years, covering taxable year 2015.

73. Date assessment was received March 08, 2016


Date petition for reinvestigation
was filed by the BIR March 18, 2016
Date of filing of documents to
support the petition April 08, 2016
Date decision of denial of the
petition was received April 28, 2016
The last day to appeal to the CTA is on:
a. April 17, 2016 c. May 8, 2016
b. April 17, 2016 d. May 28, 2016

74. Using the same facts in number 73, only that instead of going to
the CTA, a request for reconsideration was filed with the BIR on
May 7, 2016. Date decision of denial of the request for
reconsideration was received on June 2, 2016. The last day to
appeal to the CTA is on:
a. June 7, 2016 c. June 23, 2016
b. June 21, 2016 d. July 2, 2016

75. Continuing number 74 only that instead of receiving a decision


denying a request for reconsideration, a revised assessment was
received on June 2, 2016. The last day to appeal to the CTA is on:
a. June 7, 2016 c. June 23, 2016
b. June 21, 2016 d. July 2, 2016
30 days from June 2, 2016

76. Statement 1. The Bureau of Internal Revenue (BIR) has as its powers
and duties the assessment and collection of the national internal
revenue taxes, fees, and changes and the enforcement of all
fortitudes, penalties, and fines connected therewith.

Statement 2. The Bureau of Customs (BOC) has its powers and duties
the assessment and collection of awful revenues from imported and
exported articles and all other dues, fees, charges, fines, and
penalties accruing under the tariff and customs laws.
a. Both statements are correct.
b. Both statements are wrong.
c. Statement 1 is correct but statement 2 is wrong.
d. Statement 2 is correct but statement 1 is wrong.

Downloaded by jaene gallora (nichjaene05@gmail.com)


lOMoARcPSD|5612816

77. A customs duty that imposes both advance and specific customs
duties on imported or exported articles is:
a. Anti-dumping duty.
b. Countervailing duty.
c. Discriminating duty.
d. Compound customs duty.

78. Consider the following statements:


i. The imposition of customs duties also assists in economic
development.
ii. Customs duties are sometimes imposed to protect local
consumers.
iii. The purpose of regular customs duties is to raise
revenues to meet the needs of government.
iv. Compound customs duties are computed only on the basis of
units of measure, such as weight, measurement, quantity,
etc.
a. All the above statement is correct.
b. Only statements I, II and III are correct.
c. Only statements I, II and IV are correct.
d. Only statement I, III and IV are correct.

Numbers 79 to 80 are based on the following information pertaining to


taxable year 2016:
Phil. USA
Sales P500,000 P600,000
Sales discounts,
Returns and allowances 20,000 50,000
Cost of sales 230,000 250,000
Rent expenses 50,000 80,000
Salaries and wages 50,000 60,000
Interest expenses 10,000 -
Entertainment and presentation 10,000 -
Contribution to charitable institution 10,000 -
Interest on bank deposit 10,000 20,000
Royalty- musical compositions 20,000 20,000
Dividends from domestic corporation 10,000 -

79. The taxable income if the taxpayer is a domestic corporation (DC)


and resident citizen (RC), under:

Itemized Deductions Optional Deduction


DC RC DC RC
a. P320,000 P270,000 P330,000 P280,000
b. P328,150 P278,150 P354,000 P592,000
c. P280,000 P230,000 P378,000 P328,000
d. P288,150 P238,150 P330,000 P568,000

ITEMIZED DEDUCTIONS DC RC
Passive Income 40,000 40,000

Sales 1,100,000 1,100,000

Downloaded by jaene gallora (nichjaene05@gmail.com)


lOMoARcPSD|5612816

Sales Disc, Ret & Allowances (70,000) (70,000)____


Net Sales 1,030,000 1,030,000
Cost of Good Sold (480,000) (480,000)
Business Gross Income 550,000 550,000

Total Gross Income 590,000 590,000


Other Expenses (240,000) (240,000)
Interest 10,000 - (10,000 x 33%) (6,700) (6,700)
Entert & Repre (1,030,000 x .5%) (5,150) (5,150)
Net Income Before CC 338,150 338,150

CC Actual < Limit (10,000) (10,000)


Personal Exemption -0- (50,000)
Taxable Income 328,000 278,150

OPTIONAL DEDUCTION DC RC
Passive Income 40,000 40,000
Net Sales 1,030,000
Business Gross Income 550,000
Total Gross Income/
Gross Receipt & Net Sales 590,000 1,070,000
40% OSD (236,000) (428,000)
Personal Exemption -0- (50,000)
Taxable Income 354,000 592,000

80. The taxable income if the taxpayer is a resident foreign


corporation (RFC) and non-resident citizen (NRC):
RFC NRC
a. P160,000 P270,000
b. P130,900 P 80,900
c. P133,855 P 80,900
d. P120,000 P 70,000

ITEMIZED DEDUCTIONS RFC NRC


Passive Income
Sales 500,000 500,000

Sales Disc, Ret & Allowances (20,000) (20,000) _


Net Sales 480,000 480,000
Cost of Good Sold (230,000) (230,000)
Other Expenses (100,000) (100,000)
Interest 10,000 - (10,000 x 33%) (6,700) (6,700)
Entert & Repre (1,030,000 x .5%) (2,400) (2,400)
Net Income Before CC 140,900 140,900

CC 140,900 x 5% (7,045) (10,000)


Personal Exemption -0- (50,000)
Taxable Income 133,855 80,900

Downloaded by jaene gallora (nichjaene05@gmail.com)


lOMoARcPSD|5612816

TAXATION QUIZZER PART 3

1. The following are common to the inherent power of taxation, power of


eminent domain and police power, except for which of the following?
a. They are necessary attributes to the sovereignty.
b. They interfere with private rights and property.
c. They affect all persons or the public.
d. They are legislative in implementation.

2. In case of ambiguity, tax laws shall be interpreted:


a. Strictly against the taxpayer.
b. Liberally against the government.
c. Liberally in favor of the taxpayer.
d. Liberally in favor of the government.

3. Which of the following statements is not correct?


a. Taxes may be imposed to raise revenue or to provide incentives
or disincentives for certain activities within the state.
b. The state can have the power of taxation even if the
constitution does not expressly give it the power to tax.
c. For the exercise of the power of taxation, the state can tax
anything at anytime.
d. The provisions of taxation in the Philippine Constitution are
grants of power and not limitation on the taxing power.

4. Which of the following is not a direct tax?


a. Immigration tax c. Income tax
b. Transfer tax d. Contractor’s tax (Now VAT)

5. In which situation will a CPA’s signature be necessary?


a. When the value of the gross estate is P2,000,000 and above.
b. When the value of the gross estate exceeds P2,000,000.
c. Regardless of the value where the gross estate consists or
registered or registrable property.
d. When the value of the gross estate exceeds P200,000, although
exempt from transfer tax.

6. A seller of goods is non-VAT registered. His annual gross sales amount


to P1,919,500. To what business tax is he liable?
a. 3% tax on VAT-exempt persons c. 3% common carrier’s tax
b. 12% value-added tax d. Not subject to any percentage tax

7. Abigail sold through the local stock exchange, 10,000 PLDT shares that
she bought 2 years ago. Abigail sold the shares for P2 million and
realized a net gain of P200,000. The transactions is
a. Subject to regular income tax rates for individuals but only 50%
shall be recognized because it is a long-term capital gain.
b. Subject to capital gains tax amounting to P15,000.
c. Subject to percentage tax amounting to P10,000.
d. Subject to regular income tax rates for individual under Section
24 (A).

Downloaded by jaene gallora (nichjaene05@gmail.com)


lOMoARcPSD|5612816

8. Proceeds of life insurance to the extent of the amount receivable by


the estate of the deceased, his executor or administrator under
policies taken out by the dependent upon his own life shall be
I. Part of the gross estate irrespective of whether or
not the insured retained the power of revocation.
II. Not part of the gross estate if the beneficiary is
irrevocable.
III. Part of the gross income if the designation of the
beneficiary is revocable.
IV. Not part of the gross income irrespective of whether
or not the insured retained the power of revocation.
a. I and II
b. I and III
c. I and IV
d. only I

9. One of the following is not correct


Deduction Maximum
a. Funeral Expenses P200,000
b. Family home P2,000,000
c. Medical expenses P500,000
d. Standard Deduction P1,000,000

10. One of them is not considered non-resident citizen?


a. A citizen of the Philippines who establishes to the satisfaction
of the Commissioner the fact of his physical presence abroad
with a definite intention to reside therein.
b. A citizen of the Philippines who leaves the Philippines during
the taxable year to reside abroad, either as an immigrant or for
employment on permanent basis.
c. A citizen of the Philippines who works and derives income from
abroad and whose employment thereat requires him to be
physically present abroad most of the time during the taxable
year.
d. A citizen of the Philippines who went on a business trip abroad
and stayed therein most of the time during the year.

11. Z is a Filipino immigrant living in the United States for more than 10
years. He is retired and he came back to the Philippines as a
balikbayan. Every time he comes to the Philippines, he stays here for
about a month. He regularly receives a pension from his former
employer in the United States, amounting to USD1,000 a month. While in
the Philippines, with his pension pay from his former employer, he
purchased 3 condominium units in Makati, which he is renting out for
P15,000 a month each. Does the USD1,000 pension become taxable because
he is now in the Philippines?
a. Yes. Income received in the Philippines by non-resident citizens
is taxable.
b. Yes. Income received in the Philippines or abroad by non-
resident citizens is taxable.

Downloaded by jaene gallora (nichjaene05@gmail.com)


lOMoARcPSD|5612816

c. No. Income earned abroad by non-resident citizens is no longer


taxable in the Philippines.
d. No, the pension is exempt from taxation being one of the
exclusions from gross income.

12. Which of the following income earners is required to file income tax
return?
a. Minimum wage earners
b. Non-resident alien not engaged in business
c. An individual with respect to pure compensation income deriving
from such sources within the Philippines, the income tax on
which has been correctly withheld and that an individual
deriving compensation from one employer at any time during the
taxable year
d. General professional partnership

13. Which of the following fringe benefit is not subject to fringe benefit
tax?
a. a. Housing benefit c. De minimis benefit
b. b. Expense account d. Vehicle benefit

14. In the case of a taxpayer, only the following percentages of the gain
or loss recognized upon the sale or exchange of a capital asset shall
be taken into account in computing net capital gain, net capital loss
and net income.
I. Statement 1 One hundred percent (100%) if the capital asset
has been held for not more than twelve (12) months by a
taxpayer.
II. Statement 2 Fifty percent (50%) if the capital asset has been
held for more than twelve (12) months by a taxpayer.
a. True, true c. False, false
b. True, false d. False, true

15. 15.A, worked for a manufacturing firm but due to business reverses,
the firm offered a voluntary redundancy program in order to reduce
overhead expenses. Under the program, an employee who offered to
resign would be given separation pay equivalent to his 3 months basic
salary for every year of service. A accepted the offer and received
P800,000 as separation pay under the program. After all employees who
accepted the offer were paid, the firm found its overhead still
excessive. Hence, it adopted another program, where various
unprofitable departments were closed. As a result, B was separated
from the service. B also received P800,000 as separation pay. At the
time of separation both A and B have rendered at least 10 years of
service but A was 55 years old while B was only 45 years old. As a
result,
a. Both amounts are exempt from income tax
b. Both amounts are subject to income tax
c. Only Mr. A is subject to income tax
d. Only Mr. B is subject to income tax

Downloaded by jaene gallora (nichjaene05@gmail.com)


lOMoARcPSD|5612816

16. The following shall not be subject to estate tax


a. The merger of usufruct in the owner of naked title
b. The transmission or delivery of inheritance or legacy by the
fiduciary heir or legatee to fideicommissary
c. The transmission from the first heir, legatee, or donee in favor
of another beneficiary in accordance with the desire of the
predecessor
d. All bequest, devises, legacies or transfers in social welfare,
cultural and charitable institutions

17. Which of the following transactions is subject to zero-rated value-


added tax?
a. Services rendered to persons engaged in international shipping
or air transport operations
b. Services rendered banks, non-bank financial intermediaries
c. Generation, transmission and distribution of electricity
d. Services rendered by professionals such as CPA, Physicians, and
Lawyers.

18. Which of the following businesses is allowed presumptive input value-


added tax?
a. Manufacturer of canned goods
b. Manufacturer of packed juices
c. Manufacturer of packed noodles
d. Manufacturer of dried fish

19. Which of the following shall be allowed as a deduction from the gross
income?
a. Depreciation for vehicle for land transport used for personal
purposes the value of which does not exceed P2,400,000.
b. Depreciation for vehicle for land transport used in the business
the value of which exceeds P2,400,000.
c. Depreciation for land vehicles the value of which exceeds
P2,400,000 where the taxpayer’s main line of business is
transport operations or lease of transportation equipment and
the vehicles purchased are used in said operations.
d. Depreciation for airplanes and/or aircraft the value of which
exceeds P2,400,000 where the taxpayer’s main line of business is
sale of goods or properties and the vehicles purchased are used
in said operations.

20. A, at the time of retirement, had 1,000 pieces of merchandise which


was deemed sold at a value of P20,000, with an output tax of P2,400.
After retirement, A sold to B 500 pieces of these for P12,000. In the
contract of sale or invoice, A stated sales invoice number wherein the
output tax on deemed sale was imposed and the corresponding tax paid
on the 500 pieces. He prepared the following invoice:
I. Gross selling price P10,800
II. VAT previously paid on deemed sale 1,200
III. Total P12,000

Downloaded by jaene gallora (nichjaene05@gmail.com)


lOMoARcPSD|5612816

How much is the input tax of B?


a. P2,400
b. P1,200
c. P1,000
d. None of the above

21. If a VAT-registered person issues a VAT invoice or VAT official


receipt for VAT-exempt transaction, but fails to display prominently
on the invoice or receipt the words “VAT-exempt sale”, the transaction
shall:
a. Still be exempt from value-added tax
b. Become taxable and the issuer shall be liable to pay VAT thereon
c. Be effectively subject to zero percent
d. Be considered erroneous transaction and must be disregarded.

22. Which of the following franchise grantees is subject to the 2%


percentage tax on franchise?
a. Franchise on radio and/or television broadcasting companies the
gross annual receipts in the preceding year do not exceed
P10,000,000.
b. Franchise on gas and water utilities.
c. Franchise on toll road operations.
d. PAGCOR and its licenses and franchisees.

23. Which of the following is subject to value-added tax?


a. Medical, dental, hospital, and veterinary services.
b. Laboratory services.
c. Sale of drugs and medicines in the hospital’s pharmacy or drug
store.
d. Medicine administered to a patient confined in the hospital.

24. Which of the following statements is incorrect regarding standard


deduction?
a. A deduction in the amount of P1,000,000 shall be allowed as an
additional deduction without need of substantiation.
b. The full amount of P1,000,000 shall be allowed as deduction for
the benefit of the decedent.
c. Standard deduction is a deduction from the conjugal or community
properties of married decedents.
d. Standard deduction is not allowed to decedents who are non-
residents alien.

25. The actual residential home of the decedent and his family at the time
of his death, as certified by the Barangay Captain of the locality
where the family home is situated has a fair market value of
P1,500,000. The family home is part of the common property of the
spouses. How much is the family home deduction?
a. a. P1,500,000 c. P750,000
b. b. P1,000,000 d. P500,000

Downloaded by jaene gallora (nichjaene05@gmail.com)


lOMoARcPSD|5612816

26. Who makes revenue regulations?


a. Secretary of Finance c. Board of Accountancy
b. Commissioner of BIR d. House of Reperesentatives

27. Diety is non-stock, non-profit organization made an importation of


agricultural product in its original state from a Chinese farmer
amounting to P2,240,000. If you are the customs collector how will you
treat such importation, will it be?
a. a. Subject to 12% VAT c. Exemption from VAT
b. Subject to zero rated VAT d. Subject to 3% percentage tax

28. George and Pearl were legally separated. They had six minor children,
all qualified to be claimed as additional exemptions for income tax
purposes. The court awarded custody of two of the children to George
and three to Pearl, with George directed to provide full financial
support for them as well. The court awarded the 6th child to George’s
father with George also providing full financial support. Assuming
that only George is gainfully employed while Pearl is not, how much
could George claim as additional exemptions when he files his income
tax return?
a. a. P100,000 c. P50,000
b. b. P75,000 d. None

29. Construction by XYZ Construction Co. of concrete barrier for the Asian
Development Bank in Ortigas Center to prevent car bombs from ramming
the ADB gates along ADB Avenue in Mandaluyong City is subject to what
business tax?
a. a. 12% VAT c. Exemption from VAT
b. b. 0% VAT d. None of the above
30. ______________ a number of inquiries from shareholders, James Hong has
issued a formal announcement that his company is doing well.
a. Following b. Beside c. Against d. Toward

31. Mr. Araki, a non-resident alien stockholder, receives a dividend


income of P300,000 in 2012 from a foreign corporation doing business
in the Philippines. The gross income of the foreign corporation from
within and without the Philippines for three years preceding 2012 are
as follows:
2009 2010 2012
From w/in the Philippines P16,000,000 P12,000,000 P14,000,000
From without the Philippines 18,000,000 14,000,000 16,000,000

How much of the dividend income received by Mr. Araki is considered


income from sources within the Philippines?
a. Zero b. P150,000 c. P270,000 d. P300,000

Downloaded by jaene gallora (nichjaene05@gmail.com)


lOMoARcPSD|5612816

32. Martin Brian is an independent travelling salesman, deriving his


income solely from commissions and personally bearing all expenses
without reimbursement or any kind. During the taxable year 2016,
Martin Brian incurs the following expenses pertaining directly to his
activities as an independent salesman:

Travelling expenses P650,000


Secretarial expenses 250,000
Long-distance telephone bills 150,000
Freight expenses for products sold 300,000
Insurance for product transported 50,000
Life insurance premiums 25,000 do not include
Doctor’s fee incurred while he was on 15,000 do not include
one of his sales trip

What amount can Martin Brian deduct from his gross income?
a. P1,440,000
b. P1,400,000
c. P1,150,000
d. P40,000

33. A mode of acquisition by virtue of which, the property, rights and


obligations, to the extent of the value of the inheritance of a person
are transmitted through his death to another either by his will or by
operation of law
a. Extinctive prescription c. Succession
b. Acquisititive prescription d. Donation mortis causa

34. In 2016, Claudio, a self employed resident citizen, paid the


following:
Philippine income tax for 2016 P1,000,000
Professional tax 300
Gravel and sand tax 20,000
U.S.A income tax 2016 500,000
Real property tax on his house 3,000
Road user’s tax on his delivery vans 50,000
Local annual fixed tax for his delivery vans 10,000
Interest for late payment of national & local taxes 40,000
Surcharge for late payment of national & local taxes 60,000

If Claudio had signified in his return to avail of tax credit of foreign


income tax paid, how much deduction may he claim on his 2016 income?
a. P1,583,300 c. P580,300
b. P583,300 d. P80,300

35. Rose Company provided fringe benefit to its managerial employees in


the amount of P136,000 and to its rank and file employees amounting to
P50,000. The deductible expense by Rose Company is:
a. P250,000 c. P136,000
b. P186,000 d. P50,000

Downloaded by jaene gallora (nichjaene05@gmail.com)


lOMoARcPSD|5612816

36. The producers of the new play After Dusk decided to add more
performances because of the ___________ positive response to early
shows.
a. Overwhelm c. Overwhelmed
b. Overwhelming d. Overwhelmingly

37. Estate tax is


a. An excise tax c. A percentage tax
b. An income tax d. A sales tax

38. The head priest of the religious sect Tres Personas Solo Dios as the
corporation sole, rented out 5,000 sq.m. lot registered in its name
for use as school site of a school organized for profit. The sect used
the rentals for the support and upkeep of its priests. The rented lot
is
a. Not exempt from real property taxes because the user is
organized for profit.
b. Exempt from real property taxes since it is actually, directly
and exclusively used for religious purposes.
c. Not exempt from real property taxes since it is the rents not
the land that it used for religious purposes.
d. Exempt from real property taxes since it is actually, directly
and exclusively used for educational purposes.

39. Which of the following compensation of minimum wage earner shall be


taxable?
a. Holiday pay c. Night shift differential pay
b. Overtime pay d. Commission pay

40. Which principle of sound tax system suggests that the sources of
revenue should be sufficient to meet the requirements of the
government expenditures?
a. Fiscal adequacy c. Theoretical justice
b. Administrative feasibility d. Uniformity of taxes

41. The following are the powers of the Bureau of Internal Revenue,
except:
a. The assessment and collection of all national internal revenue
taxes.
b. The execution of all judgments in all cases decided in its favor
by CTA and the ordinary courts.
c. The enforcement of all forfeitures, penalties, and fines
connected with national revenue taxes.
d. The assessment and collection of customs duties and tariffs.

42. Statement 1 An individual taxpayer is allowed to claim an optional


standard deduction of 40% of gross income.
Statement 2 A corporate taxpayer is allowed to claim optional standard
deduction of 40% of gross income.
a. a. True, true b. True, false c. False, true d. False,
false

Downloaded by jaene gallora (nichjaene05@gmail.com)


lOMoARcPSD|5612816

43. Vivian is a VAT registered grocery owner and sugar dealer. She
submitted lists of inventory as of December 31, 2016 to the Revenue
District Officer as follows:
Grocery items, total value P325,000
Raw cane sugar, total value 255,000

In January 2017, she had the following sales and purchases:


Sales Purchases
Grocery (total invoice value) P1,042,525 P 650,100
Raw cane sugar (excluding VAT) 480,000 420,000

The VAT due for January 2017 is


a. P6,675.26 b. P35.675.00 c. P41,675.31 d. P42,045.54

**(1,042,525 X 12%/1.12) - (650,100 X 12%/1.12) = 42,045,54

44. Which is correct?


a. Basic personal exemption of a single individual is P20,000.
b. Basic personal exemption of a head of the family is P25,000.
c. Basic personal exemption of married individual is P32,000.
d. Additional personal exemption per qualified dependent is
P25,000.

45. This shall be considered a livelihood activity only and shall not be
considered doing business, if the gross sales or receipts in one year
does not exceed
a. P100,000 c. P200,000
b. P550,000 d. P60,000

46. Antoine Bee is the operator of Globe Bee Coliseum. During the month,
it had the following gross receipts from various activities, to wit:
I. Concert by April Boy Ireneo P85,000
II. Professional Basketball game 120,000
III. Amateur basketball games 36,700
IV. Billiards 45,000
The percentage tax payable by Antoine Bee on the Coliseum is:
a. P18,000 c. P24,750
b. P23,505 d. P43,005
120,000 x 15% = 18,000

47. As franchise. Ms. L.A. Young had the following data on revenues and
receivables, taxes not included:
Revenue A/R, beginning A/R, ending
From operations:
Covered by franchise P2,000,000 P300,000 P400,000
Not covered by the franchise 600,000 50,000

How much is the percentage tax, if operating a water system?


a. P38,000 b. P52,000 c. P90,000 d. P190,000
300,000 + 2,000,000 - 400,000 = 1,900,000 x 2% = 38,000

Downloaded by jaene gallora (nichjaene05@gmail.com)


lOMoARcPSD|5612816

Items 48 – 52 are based on the following information:


The decedent, a married man with surviving spouse under conjugal partnership
of gains, with the following data died on March 1, 2016.

Real Properties:
Family lot acquired by the decedent before marriage, FMV P 500,000

Family house built using the salary earned by the surviving


spouse during the marriage, FMV 900,000

Coconut farm inherited by the surviving spouse during the


marriage, FMV 100,000

Mango orchard, FMV 800,000

Personal Properties:
Bank deposit under the name of the decedent
representing salary earned before marriage P2,150,000

Gold necklace inherited by the surviving spouse


during the marriage 120,000

Several pieces of jewelry acquired during the marriage using


the exclusive money of the decedent 300,000

The following expenses/deductions are claimed:


Funeral expenses (40% shouldered by relatives) 100,000
Judicial expenses (including P50,000 lawyer’s fee incurred
on September 5, 2016) 120,000
A piece of jewelry lost through theft on August 15, 2016 50,000
Gambling debts of the decedent 70,000
Unpaid realty tax (coconut farm) for the 3 quarters of 2016) 150,000
Claims against an insolvent debtor (25% can be collected) 80,000
Medical expenses 600,000

48. How much is the conjugal properties?


a. P1,780,000
b. P1,700,000
c. P900,000
d. None of the choices

49. How much is the exclusive properties?


a. P2,950,000 c. P450,000
b. P650,000 d. None of the choices

50. How much is the family home deduction?


a. P1,000,000 c. P700,000
b. P950,000 d. None of the choices

51. How much is the total ordinary deductions?


a. P460,000 c. P260,000
b. P310,000 d. None of the choices

Downloaded by jaene gallora (nichjaene05@gmail.com)


lOMoARcPSD|5612816

Funeral Expense(100,000 x 60%) 60,000


Judicial Expenses 120,000
Jewelry theft 50,000
Claims against Insolvent person 80,000
Total Ordinary Deductions 310,000

52. How much is the taxable net estate?


a. a. P1,225,000 c. P1,125,000
b. b. P1,175,000 d. None of the choices

Items 53 - 56 are based on the following information:

Jon Asty, single resident citizen (who got married during the year 2016),
has the following dependents for the taxable year 2016.

Leo, adopted child, 25 years old, person with disability, dies during the
year

Dax Niel, child of his wife in a previous marriage, 18 years old

John Denver, acknowledged natural child, becomes 21 years on January 1, 2016

John Rey, child by natural adoption, 15 years old

Efren, born December 31, 2016 (taxpayer and his wife’s first born child)

The following information pertain to his income and expenses in the year
2016:
Salary, gross of P8,500, withholding tax P220,000
Rent expense, apartment house 36,000
Health insurance premium paid (P500/month starting August) 2,500
Gross sales from trading business 150,000
Cost of sales 120,000
Itemized business expenses 10,000

53. The total personal exemptions shall be:


a. a. P150,000 c. P75,000
b. b. P100,000 d. P60,000

54. The deductible health insurance premium paid is:


a. a. P2,400 c. P1,000
b. b. P1,200 d. None

55. The taxable net income using itemized deduction is:


a. a. P164,000 c. P189,000
b. b. P160,000 d. P89,000

56. The taxable net income using optional standard deduction is:
a. P164,000 c. P189,000
b. P160,000 d. P89,000

Downloaded by jaene gallora (nichjaene05@gmail.com)


lOMoARcPSD|5612816

Items 57 - 61 are based on the following information:


Joefrey Ochoa Manufacturing Corp. is a VAT-registered enterprise which is
also engaged in VAT-exempt transactions. It has the following data taken
from its books of accounts for the first quarter of the fiscal year (May 1,
2016 – April 30, 2017)

Domestic sale of goods P5,000,000


VAT-Exempt sale of goods 1,000,000
Domestic purchases of goods for use in VAT-subject transactions 500,000
Importation of goods for use in VAT-subject transactions 800,000
Domestic purchases of services for all transactions 450,000
Purchase of vehicles for land transport for use in all transactions
(estimated life 5 years, purchased May 1, 2016) 2,400,000
Maintenance expense for the vehicle for land transport 100,000
Other operating expenses 500,000
Monthly VAT paid (May & June) 300,000

57. How much is the creditable input tax?


a. a. P460,000 c. P201,000
b. b. P223,000 d. None of the choices

58. How much is the VAT payable?


a. a. P140,000 c. P387,000
b. b. P377,000 d. P99,000

59. How much is the taxable net income?


a. P3,741,000 c. P3,170,000
b. P3,530,000 d. P2,530,000

60. When shall the VAT return be filed for the first fiscal quarter?
a. August 15, 2016 c. August 25, 2016
b. August 20, 2016 d. September 29, 2016

61. When shall the income tax return be filed for the first fiscal
quarter?
a. August 25, 2016 c. September 30, 2016
b. September 29, 2016 d. None of the choices

Items 62 - 64 are based on the following information:


Jason Elcano Corporation has a soft spot for senior citizens and persons
with disability (PWDs). As such, it hires senior citizens and PWDs to work
in the company for at least six (6) months. The following data taken from
the books of accounts that are provided by the Corporation:
Gross sales P15,000,000
Cost of sales 5,000,000
Salaries of senior citizens 500,000
Salaries of PWDs 300,000
Actual amount of assistance under Adopt-A-School
program (fully deductible) 200,000
Other operating expenses 3,000,000

Downloaded by jaene gallora (nichjaene05@gmail.com)


lOMoARcPSD|5612816

During the same period the corporation also allows 20% discount to senior
citizens and PWDs who buy goods from the company. The sales are as follows
(not part of the gross sales above).
Sales to senior citizens P 800,000
Sales to PWDs 500,000
Total P1,300,000

62. How much is the total discount given?


a. a. P260,000 c. P100,000
b. b. P160,000 d. Answer not given

63. How much is the total itemized deductions including the special
itemized deductions?
a. a. P4,480,000 c. P4,175,000
b. b. P4,435,000 d. Answer not given

64. How much is the output VAT of the corporation?


a. a. P2,079,456 c. P1,828,000
b. b. P1,848,000 d. None of the choices

65. The proceeds received under a life insurance endowment contract is not
considered part of the gross income
a. If it is so stated in the life insurance endowment policy
b. If the price for the endowment policy was not fully paid
c. Where payment is made as a result of the death of the insured
d. Where the beneficiary was not the one who took out the endowment
contract

66. When shall the fringe benefits tax return be filed?


a. On or before 10th day of the month following the quarter in which
the withholding was made.
b. On or before 10th day of the month following the withholding was
made.
c. On or before 15th day of the month following the quarter in which
the withholding was made.
d. On or before 15th day of the month following the withholding was
made.

67. A, an individual, sold to B, his brother-in-law, his residential lot


with market value of P1,000,000 for P600,000. A’s cost in the lot is
P100,000. B is financially capable of buying the lot. What tax should
be imposed and collected from A as a result of the transaction?
a. Presumed capital gains tax c. Real property tax
b. Donor’s tax d. Tax on the transfer of
real property

68. Which of the following may not raise money for the government?
a. Power of taxation c. Power of eminent domain
b. Police power d. Privatization of government’s
capital assets

Downloaded by jaene gallora (nichjaene05@gmail.com)


lOMoARcPSD|5612816

69. Riyadh Motors has succeeded _____ generating positive publicity for
its new line of ecologically friendly automobiles.
a. On
b. In
c. At
d. Of

70. The auditor has _____________ to warrant a full blown forensic audit.
a. Much evidence
b. b. Many evidence
c. c. Much evidences
d. d. Many evidences

Downloaded by jaene gallora (nichjaene05@gmail.com)

You might also like